AAFP 3 Flashcards

1
Q

A 55-year-old hospitalized white male with a history of rheumatic aortic and mitral valve disease has a 3-day history of fever, back pain, and myalgias. No definite focus of infection is found on your initial examination. His WBC count is 24,000/mm3(N 4300–10,800) with 40% polymorphonuclear leukocytes and 40% band forms. The following day, two blood cultures have grown gram-positive cocci in clusters.
Until the specific organism sensitivity is known, the most appropriate antibiotic treatment would be:

A

vancomycin and gentamicin

This patient has endocarditis caused by a gram-positive coccus. Until sensitivities of the organism are known, treatment should include intravenous antibiotic coverage for Enterococcus, Streptococcus, and methicillin-sensitive and methicillin-resistant Staphylococcus. A patient who does not have a prosthetic valve should be started on vancomycin and gentamicin, with monitoring of serum levels. Enterococcus and methicillin-resistant Staphylococcus are often resistant to cephalosporins. If the organism proves to be Staphylococcus sensitive to nafcillin, the patient can be switched to a regimen of nafcillin and gentamicin.

How well did you know this?
1
Not at all
2
3
4
5
Perfectly
2
Q

A 40-year-old white female lawyer sees you for the first time. When providing a history, she describes several problems, including anxiety, sleep disorders, fatigue, persistent depressed mood, and decreased libido. These symptoms have been present for several years and are worse prior to menses, although they also occur to some degree during menses and throughout the month. Her menstrual periods are regular for the most part.
The most likely diagnosis at this time is:

A

dysthymia

Psychological disorders, including anxiety, depression, and dysthymia, are frequently confused with premenstrual syndrome (PMS), and must be ruled out before initiating therapy. Symptoms are cyclic in true PMS. The most accurate way to make the diagnosis is to have the patient keep a menstrual calendar for at least two cycles, carefully recording daily symptoms. Dysthymia consists of a pattern of ongoing, mild depressive symptoms that have been present for 2 years or more and are less severe than those of major depression. This diagnosis is consistent with the findings in the patient described here.

How well did you know this?
1
Not at all
2
3
4
5
Perfectly
3
Q

A mother brings her 2-month-old infant to the emergency department because of profuse vomiting and severe diarrhea. The infant is dehydrated, has a cardiac arrhythmia, appears to have ambiguous genitalia, and is in distress.
This presentation suggests a diagnosis of:

A

congenital adrenal hyperplasia

Congenital adrenal hyperplasia is a family of diseases caused by an inherited deficiency of any of the enzymes necessary for the biosynthesis of cortisol. In patients with the salt-losing variant, symptoms begin shortly after birth with failure to regain birth weight, progressive weight loss, and dehydration. Vomiting is prominent, and anorexia is also present. Disturbances in cardiac rate and rhythm may occur, along with cyanosis and dyspnea. In the male, various degrees of hypospadias may be seen, with or without a bifid scrotum or cryptorchidism.

How well did you know this?
1
Not at all
2
3
4
5
Perfectly
4
Q

A 62-year-old African-American female undergoes a workup for pruritus. Laboratory findings include a hematocrit of 55.0% (N 36.0–46.0) and a hemoglobin level of 18.5 g/dL (N 12.0–16.0).
What additional findings would help establish the diagnosis of polycythemia vera?

A

A platelet count >400,000/mm3

Polycythemia vera should be suspected in African-Americans or white females whose hemoglobin level is >16 g/dL or whose hematocrit is >47%. For white males, the thresholds are 18 g/dL and 52%. It should also be suspected in patients with portal vein thrombosis and splenomegaly, with or without thrombocytosis and leukocytosis. Major criteria include an increased red cell mass, a normal O2 saturation,and the presence ofsplenomegaly. Minor criteria includeelevated vitamin B 12 levels, elevated leukocyte alkaline phosphatase, a platelet count >400,000/mm3 and a WBC count >12,000/mm3 . Patients with polycythemia vera may present with gout and an elevated uric acid level, but neither is considered a criterion for the diagnosis.

How well did you know this?
1
Not at all
2
3
4
5
Perfectly
5
Q

Over the last 6 months a developmentally normal 12-year-old white female has experienced intermittent abdominal pain, which has made her quite irritable. She also complains of joint pain and general malaise. She has lost 5 kg (11 lb) and has developed an anal fissure.
What is the most likely cause of these symptoms?

A

Crohn’s disease

The most common age of onset for inflammatory bowel disease is during adolescence and young adulthood, with a second peak at 50–80 years of age. The manifestations of Crohn’s disease are somewhat dependent on the site of involvement, but systemic signs and symptoms are more common than with ulcerative colitis. Perianal disease is also common in Crohn’s disease. Irritable colon and other functional bowel disorders may mimic symptoms of Crohn’s disease, but objective findings of weight loss and anal lesions are extremely uncommon. This is also true for viral hepatitis and giardiasis. In addition, the historical and epidemiologic findings in this case are not consistent with either of these infections. Celiac disease and giardiasis can produce Crohn’s-like symptoms of diarrhea and weight loss, but are not associated with anal fissures.

How well did you know this?
1
Not at all
2
3
4
5
Perfectly
6
Q

Which one of the following is considered first-line therapy for nausea and vomiting of pregnancy?

A

Vitamin B6

A number of alternative therapies have been used for problems related to pregnancy, although vigorous studies are not always possible. For nausea and vomiting, however, vitamin B6 is considered first-line therapy, sometimes combined with doxylamine. Other measures that have been found to be somewhat useful include ginger and acupressure.

How well did you know this?
1
Not at all
2
3
4
5
Perfectly
7
Q

A 45-year-old female presents to your office because she has had a lump on her neck for the past 2 weeks. She has no recent or current respiratory symptoms, fever, weight loss, or other constitutional symptoms. She has a history of well-controlled hypertension, but is otherwise healthy. On examination you note a nontender, 2-cm, soft node in the anterior cervical chain. The remainder of the examination is unremarkable.
What would be most appropriate at this point?

A

Monitoring clinically for 4–6 weeks, then a biopsy if the node persists or enlarges

There is limited evidence to guide clinicians in the management of an isolated, enlarged cervical lymph node, even though this is a common occurrence. Evaluation and management is guided by the presence or absence of inflammation, the duration and size of the node, and associated patient symptoms. In addition, the presence of risk factors for malignancy should be taken into account.

Immediate biopsy is warranted if the patient does not have inflammatory symptoms and the lymph node is >3 cm, if the node is in the supraclavicular area, or if the patient has coexistent constitutional symptoms such as night sweats or weight loss. Immediate evaluation is also indicated if the patient has risk factors for malignancy. Treatment with antibiotics is warranted in patients who have inflammatory symptoms such as pain, erythema, fever, or a recent infection.

In a patient with no risk factors for malignancy and no concerning symptoms, monitoring the node for 4–6 weeks is recommended. If the node continues to enlarge or persists after this time, then further evaluation is indicated. This may include a biopsy or imaging with CT or ultrasonography. The utility of serial ultrasound examinations to monitor lymph nodes has not been demonstrated.

How well did you know this?
1
Not at all
2
3
4
5
Perfectly
8
Q

A 45-year-old male is seen in the emergency department with a 2-hour history of substernal chest pain. An EKG shows an ST-segment elevation of 0.3 mV in leads V4–V6.
In addition to evaluation for reperfusion therapy, which one of the following would be appropriate?

A

Oral clopidogrel (Plavix)

This patient has an ST-segment elevation myocardial infarction (STEMI). STEMI is defined as an ST-segment elevation of greater than 0.1 mV in at least two contiguous precordial or adjacent limb leads. The most important goal is to begin fibrinolysis less than 30 minutes after the first contact with the health system. The patient should be given oral clopidogrel, and should also chew 162–325 mg of aspirin.

Enteric aspirin has a delayed effect. Intravenous β-blockers such as metoprolol should not be routinely given, and warfarin is not indicated. Delaying treatment until cardiac enzyme results are available in a patient with a definite myocardial infarction is not appropriate.

How well did you know this?
1
Not at all
2
3
4
5
Perfectly
9
Q

You see a 68-year-old mechanic for a routine evaluation. He has a 2-year history of hypertension. His weight is normal and he adheres to his medication regimen. His current medications are metoprolol (Lopressor), 100 mg twice daily; olmesartan (Benicar), 40 mg/day; and hydrochlorothiazide, 25 mg/day. His serum glucose levels have always been normal, but his lipid levels are elevated.

A physical examination is unremarkable except for an enlarged prostate and a blood pressure of 150/94 mm Hg. Laboratory studies show a serum creatinine level of 1.6 mg/dL (N 0.6–1.5) and a serum potassium level of 4.9 mmol/L (N 3.5–5.0).

The patient’s record shows blood pressures ranging from 145/80 mm Hg to 148/96 mm Hg over the past year.
What would be the appropriate next step at this point?

A

Substitute furosemide (Lasix) for hydrochlorothiazide

Resistant or refractory hypertension is defined as a blood pressure ≥140/90 mm Hg, or ≥130/80 mm Hg in patients with diabetes mellitus or renal disease (i.e., with a creatinine level >1.5 mg/dL or urinary protein excretion >300 mg over 24 hours), despite adherence to treatment with full doses of at least three antihypertensive medications, including a diuretic. JNC 7 guidelines suggest adding a loop diuretic if serum creatinine is >1.5 mg/dL in patients with resistant hypertension.

How well did you know this?
1
Not at all
2
3
4
5
Perfectly
10
Q

Actinic keratoses of the skin may progress to:

A

squamous cell cancer

Actinic keratoses are scaly lesions that develop on sun-exposed skin, and are believed to be carcinoma in situ. While most actinic keratoses spontaneously regress, others progress to squamous cell cancers.

How well did you know this?
1
Not at all
2
3
4
5
Perfectly
11
Q

A 52-year-old male presents with a small nodule in his palm just proximal to the fourth metacarpophalangeal joint. It has grown larger since it first appeared, and he now has mild flexion of the finger, which he is unable to straighten. He reports that his father had similar problems with his fingers. On examination you note pitting of the skin over the nodule.
The most likely diagnosis is:

A

Dupuytren’s contracture

Dupuytren’s contracture is characterized by changes in the palmar fascia, with progressive thickening and nodule formation that can progress to a contracture of the associated finger. The fourth finger is most commonly affected. Pitting or dimpling can occur over the nodule because of the connection with the skin.

Degenerative joint disease is not associated with a palmar nodule. Trigger finger is related to the tendon, not the palmar fascia, and causes the finger to lock and release. Ganglions also affect the tendons or joints, are not located in the fascia, and are not associated with contractures. Flexor tenosynovitis, an inflammation, is associated with pain, which is not usually seen with Dupuytren’s contracture.

How well did you know this?
1
Not at all
2
3
4
5
Perfectly
12
Q

Which one of the following is NOT considered a first-line treatment for head lice?
A. Lindane 1%
B. Malathion 0.5% (Ovide)
C. Permethrin 1% (Nix)
D. Pyrethrins 0.33%/pipernyl butoxide 4% (RID)

A

Lindane 1%

Lindane’s efficacy has waned over the years and it is inconsistently ovicidal. Because of its neurotoxicity, lindane carries a black box warning and is specifically recommended only as second-line treatment by the FDA. Pyrethroid resistance is widespread, but permethrin is still considered to be a first-line treatment because of its favorable safety profile. The efficacy of malathion is attributed to its triple action with isopropyl alcohol and terpineol, likely making this a resistance-breaking formulation. The probability of simultaneously developing resistance to all three substances is small. Malathion is both ovicidal and pediculicidal.

How well did you know this?
1
Not at all
2
3
4
5
Perfectly
13
Q

What food is a frequent cause of cross-reactive food-allergy symptoms in latex-allergic individuals?

A

Avocadoes

The majority of patients who are latex-allergic are believed to develop IgE antibodies that cross-react with some proteins in plant-derived foods. These food antigens do not survive the digestive process, and thus lack the capacity to sensitize after oral ingestion in the traditional food-allergy pathway. Antigenic similarity with proteins present in latex, to which an individual has already been sensitized, results in an indirect allergic response limited to the exposure that occurs prior to alteration by digestion, localized primarily in and around the oral cavity. The frequent association with certain fruits has been labeled the “latex-fruit syndrome.” Although many fruits and vegetables have been implicated, fruits most commonly linked to this problem are bananas, avocadoes, and kiwi.

How well did you know this?
1
Not at all
2
3
4
5
Perfectly
14
Q

A 42-year-old female is found to have a thyroid nodule during her annual physical examination. Her TSH level is normal. Ultrasonography of her thyroid gland shows a solitary nodule measuring 1.2 cm.
What is the most appropriate next step at this point?

A

A fine-needle aspiration biopsy of the nodule

All patients who are found to have a thyroid nodule on a physical examination should have their TSH measured. Patients with a suppressed TSH should be evaluated with a radionuclide thyroid scan; nodules that are “hot” (show increased isotope uptake) are almost never malignant and fine-needle aspiration biopsy is not needed. For all other nodules, the next step in the workup is a fine-needle aspiration biopsy to determine whether the lesion is malignant (SOR B).

How well did you know this?
1
Not at all
2
3
4
5
Perfectly
15
Q

Treatment with donepezil (Aricept) is associated with an increased risk for :

A

bradycardia requiring pacemaker implantation

A large population study has established a significant increased risk of bradycardia, syncope, and pacemaker therapy with cholinesterase inhibitor therapy. Elevation of liver enzymes with the potential for hepatic dysfunction has been seen with tacrine, but it has not been noted with the other approved cholinesterase inhibitors. Cataract formation and thrombosis with pulmonary embolism do not increase with this therapy. Although improvement in mental function is often marginal with cholinesterase inhibitor therapy, the therapy has not been shown to increase the need for institutionalization.

How well did you know this?
1
Not at all
2
3
4
5
Perfectly
16
Q

An 8-year-old female is brought to your office with a 3-day history of bilateral knee pain. She has had no associated upper respiratory symptoms. On examination she is afebrile. Her knees have full range of motion and no effusion, but she has a purpuric papular rash on both lower extremities.
What is the most likely cause of her symptoms?

A

Henoch-Schönlein purpura

The combination of arthritis with a typical palpable purpuric rash is consistent with a diagnosis of Henoch-Schönlein purpura. This most often occurs in children from 2 to 8 years old. Arthritis is present in about two-thirds of those affected. Gastrointestinal and renal involvement are also common.
Rocky Mountain spotted fever presents with a rash, but arthralgias are not typical. These patients are usually sick with a fever and headache. Juvenile rheumatoid arthritis is associated with a salmon-pink maculopapular rash, but not purpura. The rash associated with Lyme disease is erythema migrans, which is a bull’s-eye lesion at the site of a tick bite. The rash associated with rheumatic fever is erythema marginatum, which is a pink, raised, macular rash with sharply demarcated borders.

How well did you know this?
1
Not at all
2
3
4
5
Perfectly
17
Q

What hospitalized patients are the most appropriate candidate for thromboembolism prophylaxis with enoxaparin (Lovenox)?

A

example: A 67-year-old female with hemiparesis, admitted for community-acquired pneumonia

Venous thromboembolism is a frequent cause of preventable death and illness in hospitalized patients. Approximately 10%–15% of high-risk patients who do not receive prophylaxis develop venous thrombosis. Pulmonary embolism is thought to be associated with 5%–10% of deaths in hospitalized patients. Anticoagulant prophylaxis significantly reduces the risk of pulmonary embolism and should be used in all high-risk patients.

Prophylaxis is generally recommended for patients over the age of 40 who have limited mobility for 3 days or more and have at least one of the following risk factors: acute infectious disease, New York Heart Association class III or IV heart failure, acute myocardial infarction, acute respiratory disease, stroke, rheumatic disease, inflammatory bowel disease, previous venous thromboembolism, older age (especially >75 years), recent surgery or trauma, immobility or paresis, obesity (BMI >30 kg/m2), central venouscatheterization, inherited or acquired thrombophilic disorders, varicose veins, or estrogen therapy.

Pharmacologic therapy with an anticoagulant such as enoxaparin is clearly indicated in the 67-year-old who has limited mobility secondary to hemiparesis and is being admitted for an acute infectious disease. The patient on chronic anticoagulation, the patient with severe thrombocytopenia, and the patient with coagulopathy are at high risk for bleeding if given anticoagulants, and are better candidates for nonpharmacologic therapies such as foot extension exercises, graduated compression stockings, or pneumatic compression devices. Although the 22-year-old is obese and recently had surgery, his young age and ambulatory status make anticoagulant prophylaxis less necessary.

How well did you know this?
1
Not at all
2
3
4
5
Perfectly
18
Q

A 25-year-old white male who has a poorly controlled major seizure disorder and a 6-week history of recurrent fever, anorexia, and persistent, productive coughing visits your office. On physical examination he is noted to have a temperature of 38.3°C (101.0°F), a respiratory rate of 16/min, gingival hyperplasia, and a fetid odor to his breath. Auscultation of the lungs reveals rales in the mid-portion of the right lung posteriorly.
What is most likely to be found on a chest radiograph?

A

A lung abscess

Anaerobic lung abscesses are most often found in a person predisposed to aspiration who complains of a productive cough associated with fever, anorexia, and weakness. Physical examination usually reveals poor dental hygiene, a fetid odor to the breath and sputum, rales, and pulmonary findings consistent with consolidation. Patients who have sarcoidosis usually do not have a productive cough and have bilateral physical findings. A persistent productive cough is not a striking finding in disseminated tuberculosis, which would be suggested by miliary calcifications on a chest film. The clinical presentation and physical findings are not consistent with a simple mass in the right hilum nor with a right pleural effusion.

How well did you know this?
1
Not at all
2
3
4
5
Perfectly
19
Q

What medication should be given intravenously in the initial treatment of status epilepticus?

A

Lorazepam (Ativan)

Status epilepticus refers to continuous seizures or repetitive, discrete seizures with impaired consciousness in the interictal period. It is an emergency and must be treated immediately, since cardiopulmonary dysfunction, hyperthermia, and metabolic derangement can develop, leading to irreversible neuronal damage. Lorazepam, 0.1–0.15 mg/kg intravenously, should be given as anticonvulsant therapy after cardiopulmonary resuscitation. This is followed by phenytoin, given via a dedicated peripheral intravenous line. Fosphenytoin, midazolam, or phenobarbital can be used if there is no response to lorazepam.
Propofol has been used for refractory status epilepticus to induce general anesthesia when the initial drugs have failed, but reports of fatal propofol infusion syndrome have led to a decline in its use.

How well did you know this?
1
Not at all
2
3
4
5
Perfectly
20
Q

According to JNC 7, the risk of cardiovascular disease begins to increase when the systolic blood pressure exceeds a threshold of :

A

115 mm Hg

According to JNC 7, the risk of both ischemic heart disease and stroke increases progressively when systolic blood pressure exceeds 115 mm Hg and diastolic blood pressure exceeds 75 mm Hg.

How well did you know this?
1
Not at all
2
3
4
5
Perfectly
21
Q

A 56-year-old female has been on combined continuous hormone therapy for 6 years. This is associated with a reduced risk for ?

A

Bone fracture

Hormone replacement therapy that includes estrogen has been shown to decrease osteoporosis and bone fracture risk. The risk for colorectal cancer also is reduced after 5 years of estrogen use. The risk for myocardial infarction, stroke, breast cancer, and venous thromboembolism increases with long-term use.

How well did you know this?
1
Not at all
2
3
4
5
Perfectly
22
Q

A 2-week-old female is brought to the office for a well child visit. The physical examination is completely normal except for a clunking sensation and feeling of movement when adducting the hip and applying posterior pressure. Which one of the following would be the most appropriate next step?

A

Referral for orthopedic consultation

Developmental dysplasia of the hip encompasses both subluxation and dislocation of the newborn hip, as well as anatomic abnormalities. It is more common in firstborns, females, breech presentations, oligohydramnios, and patients with a family history of developmental dysplasia. Experts are divided with regard to whether hip subluxation can be merely observed during the newborn period, but if there is any question of a hip problem on examination by 2 weeks of age, the recommendation is to refer to a specialist for further testing and treatment. Studies show that these problems disappear by 1 week of age in 60% of cases, and by 2 months of age in 90% of cases. Triple diapering should not be used because it puts the hip joint in the wrong position and may aggravate the problem. Plain radiographs may be helpful after 4-6 months of age, but prior to that time the ossification centers are too immature to be seen. Because the condition can be difficult to diagnose, and can result in significant problems, the current recommendation is to treat all children with developmental dysplasia of the hip. Closed reduction and immobilization in a Pavlik harness, with ultrasonography of the hip to ensure proper positioning, is the treatment of choice until 6 months of age. The American Academy of Pediatrics recommends ultrasound screening at 6 weeks for breech girls, breech boys (optional), and girls with a positive family history of developmental dysplasia of the hip. Other countries have recommended universal screening, but a review of the literature has not shown that the benefits of early diagnosis through universal screening outweigh the risks and potential problems of overtreating.

How well did you know this?
1
Not at all
2
3
4
5
Perfectly
23
Q

A 55-year-old overweight male presents with a complaint of pain in the left big toe. He recently started jogging 2 miles a day to try to lose weight, but has not changed his diet and says he drinks 4 cans of beer every night. The pain has developed gradually over the last 2 weeks and is worse after running. An examination shows a normal foot with tenderness and swelling of the medial plantar aspect of the left first metatarsophalangeal joint. Passive dorsiflexion of the toe causes pain in that area. Plantar flexion produces no discomfort, and no numbness can be appreciated. Which one of the following is the most likely diagnosis?

A

Sesamoid fracture

Pain involving the big toe is a common problem. The first metatarsophalangeal (MTP) joint has two sesamoid bones, and injuries to these bones account for 12% of big-toe injuries. Overuse, a sharp blow, and sudden dorsiflexion are the most common mechanisms of injury. Gout commonly involves the first MTP joint, but the onset is sudden, with warmth, redness, and swelling, and pain on movement of the joint is common. Morton’s neuroma commonly occurs between the third and fourth toes, causes numbness involving the digital nerve in the area, and usually is caused by the nerve being pinched between metatarsal heads in the center of the foot. Cellulitis of the foot is common, and can result from inoculation through a subtle crack in the skin. However, there would be redness and swelling, and the process is usually more generalized. Sesamoiditis is often hard to differentiate from a true sesamoid fracture. Radiographs should be obtained, but at times they are nondiagnostic. Treatment, fortunately, is similar, unless the fracture is open or widely displaced. Limiting weight bearing and flexion to control discomfort is the first step. More complex treatments may be needed if the problem does not resolve in 4-6 weeks.

How well did you know this?
1
Not at all
2
3
4
5
Perfectly
24
Q

A mother brings in her 2-month-old infant for a routine checkup. The baby is exclusively breastfed, and the mother has no concerns or questions. Which one of the following would you recommend at this time in addition to continued breastfeeding?

A

Vitamin D supplementation

Although breast milk is the ideal source of nutrition for healthy term infants, supplementation with 200 IU/day of vitamin D is recommended beginning at 2 months of age and continuing until the child is consuming at least 500 mL/day of formula or milk containing vitamin D (SOR B). The purpose of supplementation is to prevent rickets. Unless the baby is anemic or has other deficiencies, neither iron nor a multivitamin is necessary. Parents often mistakenly think babies need additional water, which can be harmful because it decreases milk intake and can cause electrolyte disturbances. Cereal should not be started until 4 to 6 months of age.

How well did you know this?
1
Not at all
2
3
4
5
Perfectly
25
Q

A 62-year-old white male complains of fatigue and proximal extremity discomfort without any localized joint pain. Which condition is associated with a consistently normal creatine kinase enzyme level at all phases of disease?

A

Polymyalgia rheumatica (PMR)

Polymyalgia rheumatica is a disease of the middle-aged and elderly. Discomfort is common in the neck, shoulders, and hip girdle areas. There is an absence of objective joint swelling, and findings tend to be symmetric. Characteristically, the erythrocyte sedimentation rate and C-reactive protein levels are significantly elevated; however, these tests are nonspecific. Occasionally there are mild elevations of liver enzymes, but muscle enzymes, including creatine kinase, are not elevated in this disorder. Elevation of muscle enzymes strongly suggests another diagnosis. Polymyositis and dermatomyositis are associated with variable levels of muscle enzyme elevations during the active phases of the disease. Drug-induced myopathies such as those seen with the cholesterol-lowering statin medications tend to produce some elevation of muscle enzymes during the course of the disorder. Hypothyroidism is associated with creatine kinase elevation. It should be strongly considered in the patient with unexplained, otherwise asymptomatic creatine kinase elevation found on a routine chemistry profile. Hyperthyroidism may cause muscle disease and loss of muscle, but it is not associated with creatine kinase elevation.

How well did you know this?
1
Not at all
2
3
4
5
Perfectly
26
Q

What are the risk factors for osteoarthritis of the hip?

A

Risk factors for osteoarthritis of the hip include obesity, high bone mass, old age, participation in weight-bearing sports, and hypothyroidism.

How well did you know this?
1
Not at all
2
3
4
5
Perfectly
27
Q

The daily intake of vitamins and minerals recommended by the Food and Nutrition Board varies according to sex, age, and condition. The recommended daily allowance of vitamin D is greatest for which patient population?

A

A 75-year-old female

The current Dietary Reference Intake (DRI – which has replaced RDA’s) recommendation for vitamin D is 200 IU/day for all women between the ages of 9 and 50 years; pregnancy or lactation does not affect the recommendation. The DRI doubles to 400 IU daily for women age 51-70 and triples to 600 IU daily for women over the age of 70. The maximum daily oral intake of vitamin D thought to be safe is 2000 IU/day for all females over the age of 12 months.

How well did you know this?
1
Not at all
2
3
4
5
Perfectly
28
Q

In addition to calcium and vitamin D supplementation, patients who are beginning long-term treatment with prednisone (≥3 months at a dosage ≥5 mg/day) should also receive

A

bisphosphonate therapy

The American College of Rheumatology recommends that patients who are beginning long-term treatment with prednisone (≥3 months at a dosage ≥5 mg/day), or an equivalent, receive bisphosphonate therapy in addition to calcium and vitamin D supplementation, regardless of their DEXA-scan T score. The other treatments are not recommended for prevention of glucocorticoid-induced osteoporosis.

How well did you know this?
1
Not at all
2
3
4
5
Perfectly
29
Q

A 79-year-old male is admitted to the hospital because of a sudden inability to ambulate. He has a past history of gout. On examination his temperature is 38.2°C (100.8°F) and he has bilateral knee effusions. His WBC count is 14,000/mm3 with 82% segs. His serum uric acid level is 8.5 mg/dL (N

A

Arthrocentesis

Polyarticular arthritis often presents with fever, knee and other joint effusions, and leukocytosis. A 24-hour urine collection is not routine, is difficult for the patient, and typically does not change therapy. Especially in cases where a joint effusion is accompanied by fever, diagnostic arthrocentesis should be performed to help guide therapy. Allopurinol should not be initiated during an acute gouty attack, but may be started after a patient has recovered. Diuretics increase uric acid levels.

How well did you know this?
1
Not at all
2
3
4
5
Perfectly
30
Q

An overweight 13-year-old male presents with a 3-week history of right lower thigh pain. He first noticed the pain when jumping while playing basketball, but now it is present even when he is just walking. On examination he can bear his full weight without an obvious limp. There is no localized tenderness, and the patella tracks normally without subluxation. Internal rotation of the hip is limited on the right side compared to the left. Based on the examination alone, which one of the following is the most likely diagnosis?

A

Slipped capital femoral epiphysis

This is a classic presentation for slipped capital femoral epiphysis (SCFE) in an adolescent male who has probably had a recent growth spurt. Pain with activity is the most common presenting symptom, as opposed to the nighttime pain that is typical of malignancy. Obese males are affected more often. The pain is typically in the anterior thigh, but in a high percentage of patients the pain may be referred to the knee, lower leg, or foot. Limited internal rotation of the hip, especially with the hip in 90°; flexion, is a reliable and specific finding for SCFE and should be looked for in all adolescents with hip, thigh, or knee pain. Meralgia paresthetica is pain in the thigh related to entrapment of the lateral femoral cutaneous nerve, often attributed to excessively tight clothing. Legg-Calvé-Perthes disease (avascular or aseptic necrosis of the femoral head) is more likely to occur between the ages of 4 and 8 years. Juvenile rheumatoid arthritis typically is associated with other constitutional symptoms including stiffness, fever, and pain in at least one other joint, with the pain not necessarily associated with activity.

How well did you know this?
1
Not at all
2
3
4
5
Perfectly
31
Q

A 70-year-old female consults you about osteoporosis treatment. Two years ago her DEXA scan T score was -2.6, and she began taking risedronate (Actonel), 35 mg/week. Her BMI is 24 kg/m2, she takes appropriate doses of calcium and vitamin D, and she takes walks almost every day. Her current T score is -2.5, and she is concerned about the minimal change in spite of therapy. She has never had a fracture, but asks if more could be done to reduce her fracture risk. Which one of the following would be the most appropriate recommendation?

A

continue current therapy

There is not a linear correlation between bone mineral density and fracture risk. Bone architecture may be changed by bisphosphonate therapy, which may result in a decreased fracture risk. This patient has not had a fracture and is on adequate medical therapy that should be continued.

How well did you know this?
1
Not at all
2
3
4
5
Perfectly
32
Q

A 36-year-old male presents with pain over the lumbar paraspinal muscles. He says the pain began suddenly while he was shoveling snow. What management recommendations would you give regarding this patients injury?

A

Continued activity rather than bed rest helps speed recovery

Multiple studies have demonstrated that bed rest is detrimental to recovery from low back pain. Patients should be encouraged to remain as active as possible. Exercises designed specifically for the treatment of low back pain have not been shown to be helpful. Neither opioids nor trigger-point injections have shown superiority over placebo, NSAIDs, or acetaminophen in relieving acute back pain. There is no good evidence to suggest that systemic corticosteroids are effective for low back pain with or without sciatica.

How well did you know this?
1
Not at all
2
3
4
5
Perfectly
33
Q

A 44-year-old African-American female reports diffuse aching, especially in her upper legs and shoulders. The aching has increased, and she now has trouble going up and down stairs because of weakness. She has no visual symptoms, and a neurologic examination is normal except for proximal muscle weakness. Laboratory tests reveal elevated levels of serum creatine kinase and aldolase. Her symptoms improve significantly when she is treated with corticosteroids. Which one of the following is the most likely diagnosis?

A

Polymyositis

The patient described has an inflammatory myopathy of the polymyositis/dermatomyositis group. Proximal muscle involvement and elevation of serum muscle enzymes such as creatine kinase and aldolase are characteristic. Corticosteroids are the accepted treatment of choice. It is extremely unlikely that Duchenne’s muscular dystrophy would present after age 30. In amyotrophic lateral sclerosis, an abnormal neurologic examination with findings of upper motor neuron dysfunction is characteristic. Patients with myasthenia gravis characteristically have optic involvement, often presenting as diplopia. The predominant symptom of aseptic necrosis of the femoral head is pain rather than proximal muscle weakness.

How well did you know this?
1
Not at all
2
3
4
5
Perfectly
34
Q

You see a 5-year-old white female with in-toeing due to excessive femoral anteversion. She is otherwise normal and healthy, and her mobility is unimpaired. Her parents are greatly concerned with the cosmetic appearance and possible future disability, and request that she be treated. You recommend which one of the following?

A

Observation

There is little evidence that femoral anteversion causes long-term functional problems. Studies have shown that shoe wedges, torque heels, and twister cable splints are not effective. Surgery should be reserved for children 8-10 years of age who still have cosmetically unacceptable, dysfunctional gaits. Major complications of surgery occur in approximately 15% of cases, and can include residual in-toeing, out-toeing, avascular necrosis of the femoral head, osteomyelitis, fracture, valgus deformity, and loss of position. Thus, observation alone is appropriate treatment for a 5-year-old with uncomplicated anteversion.

How well did you know this?
1
Not at all
2
3
4
5
Perfectly
35
Q

Which medication used to treat rheumatoid arthritis can delay the progression of the disease?

A

Hydroxychloroquine (Plaquenil)

Hydroxychloroquine, originally developed as an antimalarial drug, is a well-known disease-modifying agent that can slow the progression of rheumatoid arthritis. Aspirin, indomethacin, and ibuprofen are anti-inflammatory agents. They relieve pain and improve mobility, but do not alter the progression of the disease. Capsaicin, a topical substance-P depleter, can relieve pain symptoms.

How well did you know this?
1
Not at all
2
3
4
5
Perfectly
36
Q

A 75-year-old white female presents with severe pain of the carpometacarpal joint at the base of her thumb. Examination of her hands also reveals hypertrophic changes of the distal interphalangeal and proximal interphalangeal joints of her fingers. These findings are most consistent with

A

osteoarthritis

Osteoarthritis causes changes predominantly in the proximal interphalangeal (PIP) and distal interphalangeal (DIP) joints of the hands known as Bouchard’s and Heberden’s nodes respectively, and the carpometacarpal joints of the thumbs. While rheumatoid arthritis commonly causes subluxations in the metacarpophalangeal joints, this patient’s hypertrophic changes are most likely due to osteoarthritis. The other choices are less likely to cause this presentation.

How well did you know this?
1
Not at all
2
3
4
5
Perfectly
37
Q

A 60-year-old male presents with an acute onset of pain and swelling in the right big toe. He can recall no mechanism of injury. He has hypertension which is well controlled with hydrochlorothiazide. On examination the area around the base of the toe is reddened, slightly warm, and very tender on palpation. Which medication should be AVOIDED in this patient at this time?

A

Allopurinol (Zyloprim)

This patient likely has gout. Aspiration should be attempted to get a specific diagnosis. The initial treatment for gout is NSAIDs, colchicine, or cortisone injections (SOR B). Allopurinol should be avoided until the episode of gout is controlled, because it may cause temporary worsening. In addition to medication, recommended management includes addressing risk factors such as obesity, diuretic use, high-purine diet, and alcohol intake (SOR B).

How well did you know this?
1
Not at all
2
3
4
5
Perfectly
38
Q

A 55-year-old African-American male with osteoarthritis of the knees asks for advice on improving the function of his knees and controlling arthritis pain. Which one of the following would be appropriate advice?

A

A therapeutic exercise program will improve both pain and function

A therapeutic exercise program will reduce both pain and disability in patients with osteoarthritis of the knee (SOR A). There is no evidence to support the use of capsaicin cream, but NSAIDs will reduce pain and there are proven therapies that will improve function of the patients knee. While intra-articular corticosteroids are effective in relieving pain in the short term (up to 4 weeks), there is no evidence for long-term efficacy. There is not good evidence to support the use of glucosamine for treating osteoarthritis of the knee. One systematic review found it no more effective than placebo.

How well did you know this?
1
Not at all
2
3
4
5
Perfectly
39
Q

An 83-year-old female presents with pain, swelling, and erythema of her left knee. She first noticed this problem last night before going to bed. She is generally healthy and takes no medications. She has not been sexually active since being widowed 15 years ago, and she currently lives with her sister. She states that she developed pain and swelling in her left ankle 2 years ago that lasted only a couple of days and resolved spontaneously. Blood testing shows a very elevated erythrocyte sedimentation rate but a normal rheumatoid factor and uric acid level. What is the most likely diagnosis?

A

Pseudogout (calcium pyrophosphate disease)

Acute monoarthritis in adults is most commonly caused by infection, trauma, or crystal deposition. Rheumatoid arthritis seldom presents as monoarthritis, and more often has a subacute course with multiple, symmetric joints involved. Although osteoporosis may result in a fracture of the knee joint without trauma at this age, there is no reason to believe that this patient has a torn anterior cruciate ligament. Gonococcal arthritis is one of the most common causes of septic arthritis, but is highly unlikely in this elderly, sexually inactive patient. Nongonococcal septic arthritis (especially due to staphylococcal and streptococcal bacteria) is still a consideration and should be ruled out by aspiration of fluid to be sent for culture. This patient’s presentation is most consistent with pseudogout. Having a normal uric acid level suggests against gout, but does not rule it out. Also, gout is seven times more likely to be seen in males, whereas pseudogout is 1.5 times more frequent in females. Pseudogout most often affects the elderly, and usually affects the knee, wrist, and ankle. Gout presents most commonly in the first metatarsophalangeal joint and insteps of the feet, but also can occur in the knee, wrist, finger, and olecranon bursa. Differentiating between gout and pseudogout can be difficult and is best done by analysis of joint fluid. In patients with gout, this fluid contains highly negative birefringent, needle-shaped urate crystals, whereas in pseudogout the fluid contains rhomboid-shaped, weakly positive birefringent calcium pyrophosphate crystals.

How well did you know this?
1
Not at all
2
3
4
5
Perfectly
40
Q

A 35-year-old white female comes to your office with a 3-month history of the gradual onset of pain and tenderness in the wrists and hands. She also complains of 1 hour of morning stiffness. She denies rash, fever, or skin changes. On physical examination she has symmetric swelling of the proximal interphalangeal joints and metacarpophalangeal joints. Motion of these joints is painful. She has no rash or mouth ulcers. Radiographs of the hands and wrists are negative, and a chest film is unremarkable. Her CBC is normal, but the erythrocyte sedimentation rate is elevated at 40 mm/hr. Latex fixation for rheumatoid factor is negative, and an antinuclear antibody (ANA) test is negative. The most likely diagnosis in this patient is:

A

Rheumatoid arthritis

This patient has rheumatoid arthritis (RA) by symptoms and physical findings. A positive latex fixation test for rheumatoid factor is not necessary for the diagnosis. A negative rheumatoid factor does not exclude RA, and a positive rheumatoid factor is not specific. Rheumatoid factor is found in the serum of approximately 85% of adult patients with RA; in subjects without RA, the incidence of positive rheumatoid factor is 1%–5% and increases with age. The ANA test is positive in at least 95% of patients with systemic lupus erythematosus (SLE), but in only about 35% of patients with RA. Elevation of the erythrocyte sedimentation rate is seen in many patients with RA, and the degree of elevation roughly parallels disease activity. A mean of 6 months after the onset of Lyme disease, 60% of patients in the U.S. have brief attacks of asymmetric, oligoarticular arthritis, primarily in the large joints and especially in the knee.

How well did you know this?
1
Not at all
2
3
4
5
Perfectly
41
Q

A 40-year-old runner complains of gradually worsening pain on the lateral aspect of his foot. He runs on asphalt, and has increased his mileage from 2 miles/day to 5 miles/day over the last 2 weeks. Palpation causes pain over the lateral 5th metatarsal. The pain is also reproduced when he jumps on the affected leg. When you ask about his shoes he tells you he bought them several years ago. What is the most likely diagnosis?

A

Stress fracture

Running injuries are primarily caused by overuse due to training errors. Runners should be instructed to increase their mileage gradually, in increments of 10% or less each week. A stress fracture causes localized tenderness and swelling in superficial bones. Pain is reproduced by jumping on the affected leg. Plantar fasciitis causes burning pain in the heel and there is tenderness of the plantar fascia where it inserts onto the medial tubercle of the calcaneus.

How well did you know this?
1
Not at all
2
3
4
5
Perfectly
42
Q

What clinical presentation is most consistent with vitamin D deficiency in the aged?

A

Development of chronic bone pain and weakness in association with bone loss

Vitamin D deficiency is being recognized more frequently among the elderly, especially in the nursing home or other settings where inactivity, nutritional deficiency, and lack of sunlight exposure combine to increase the propensity for deficiency. Vitamin D deficiency has been associated with abnormalities in bone metabolism, primarily osteomalacia. Clinically this can present as muscle weakness, limb pain, and impaired physical function. Bleeding gums and fatigue may be presenting symptoms of scurvy (vitamin C deficiency). High-output cardiac failure with vasodilation, dermatitis, and neuropathy is the clinical presentation of thiamine deficiency. The combination of diarrhea, scaly dermatitis, and dementia is one of the clinical presentations of niacin deficiency.

How well did you know this?
1
Not at all
2
3
4
5
Perfectly
43
Q

An anxious and agitated 18-year-old white male presents to your office with a 2-hour history of severe muscle spasms in the neck and back. He was seen 2 days ago in a local emergency department with symptoms of gastroenteritis, treated with intravenous fluids, and sent home with a prescription for prochlorperazine (Compazine) suppositories. The best therapy for this problem is intravenous administration of:

A

Diphenhydramine (Benadryl)

While rarely life threatening, an acute dystonic reaction can be frightening and painful to the patient and confusing to the treating physician who may be unaware of what medications the patient is taking. Dystonia can be caused by any agent that blocks dopamine, including prochlorperazine, metoclopramide, and typical neuroleptic agents such as haloperidol. The acute treatment of choice is diphenhydramine or benztropine.

How well did you know this?
1
Not at all
2
3
4
5
Perfectly
44
Q

For several years, a hypertensive 65-year-old female has been treated with hydrochlorothiazide (HydroDIURIL), 25 mg/day; atenolol (Tenormin), 100 mg/day; and hydralazine (Apresoline), 50 mg 4 times/day. Her blood pressure has been well controlled on this regimen. Over the past 2 months she has experienced malaise, along with diffuse joint pains that involve symmetric sites in the fingers, hands, elbows, and knees. A pleural friction rub is noted on examination. Laboratory testing shows that the patient has mild anemia and leukopenia, with a negative rheumatoid factor and a positive antinuclear antibody (ANA) titer of 1:640. What would be the most appropriate INITIAL treatment?

A

Discontinue the hydralazine

There are many drugs that can induce a syndrome resembling systemic lupus erythematosus, but the most common offender is procainamide, followed by hydralazine. There is a genetic predisposition for this drug-induced lupus, determined by drug acetylation rates. Polyarthritis and pleuropericarditis occur in half of patients, but fortunately, CNS and renal involvement are rare. While all patients with this condition have positive ANAs and most have antibodies to histones, antibodies to double-stranded DNA and decreased complement levels are rare, which distinguishes drug-induced lupus from idiopathic lupus. The best initial management is to withdraw the drug, and most patients improve in a few weeks. For those with severe symptoms, a short course of corticosteroids is indicated. Once the offending drug is discontinued, symptoms seldom last beyond 6 months

How well did you know this?
1
Not at all
2
3
4
5
Perfectly
45
Q

A 37-year-old white female who has had silicone breast implants for 17 years is concerned about the risk of developing joint problems from the implants. You discuss studies concerning risks of connective tissue disease in women who have silicone breast implants and assure her that

A

Assure her that there is no increased risk of connective tissue disease in women with silicone implants

Despite widespread publicity and legal wrangling, a large meta-analysis of women who have had silicone breast implants has produced no evidence of any significant increase in the risk for connective tissue disease in women who have these implants.

How well did you know this?
1
Not at all
2
3
4
5
Perfectly
46
Q
A high incidence of osteonecrosis of the femoral head occurs with:  
 A. Femoral neck fractures 
 B. Intertrochanteric fractures 
 C. Subtrochanteric fractures 
 D. Femoral shaft fractures 
 E. Supracondylar femoral fractures
A

Femoral neck fractures

Femoral neck fractures disrupt the blood supply to the femoral head, which can lead to osteonecrosis. This does not occur with the other types of fractures.

How well did you know this?
1
Not at all
2
3
4
5
Perfectly
47
Q

A 3-year-old white female is brought to your office because she is complaining of pain in her right arm. Her mother tells you the pain began after she pulled her daughter by the arm while the girl was fighting with her brother. You examine the child and diagnose “nursemaid’s elbow.” You recommend which intervention?

A

Manipulation of the forearm to reduce radial head subluxation

Subluxation of the radial head (nursemaid’s elbow) is a common childhood orthopedic problem. There is conflicting information in various textbooks on the proper technique to reduce the subluxed radial head. However, a study comparing hyperpronation to supination/flexion found that hyperpronation had a higher success rate. If a fracture is diagnosed, then a sling would be helpful. Neither a wrist splint nor a long arm cast is helpful in this situation.

How well did you know this?
1
Not at all
2
3
4
5
Perfectly
48
Q

A 91-year-old white male presents with a 6-month history of a painless ulcer on the dorsum of the proximal interphalangeal joint of the second toe. Examination reveals a hallus valgus and a rigid hammer toe of the second digit. His foot has mild to moderate atrophic skin changes and the dorsal and posterior tibial pulses are absent. Appropriate treatment includes:

A

Custom-made shoes to protect the hammer toe

The treatment of foot problems in the elderly is difficult because of systemic and local infirmities, the most limiting being the poor vascular status of the foot. Conservative, supportive, and palliative therapy replaces definitive reconstructive surgical therapy. Surgical correction of a hammer toe and bunionectomy would be disastrous in an elderly patient with a small ulcer and peripheral vascular disease. The correct approach to this patient is to prescribe custom-made shoes and a protective shield with a central aperture of foam rubber placed over the hammer toe. Metatarsal pads are not useful in the treatment of hallux valgus and a rigid hammer toe

How well did you know this?
1
Not at all
2
3
4
5
Perfectly
49
Q

A 67-year-old female is concerned about osteoporosis. The study of choice for determining whether or not she has osteoporosis is:

A

Dual-energy x-ray absorptiometry (DEXA) of the hip and spine

Dual-energy x-ray absorptiometry (DEXA) of the hip and spine is the method of choice for assessment of bone mineral density. Quantitative CT is the most sensitive method, but results in substantially greater radiation exposure. Plain radiographs are not sensitive enough to diagnose osteoporosis until total bone density has decreased by 50%. The predictive value of DEXA scans and ultrasound examinations of peripheral bones in assessing fracture risk at the hip or vertebrae is not clear.

How well did you know this?
1
Not at all
2
3
4
5
Perfectly
50
Q
Which one of the following is consistent with spinal stenosis but not herniated vertebral disk? 
 A. Numbness 
 B. Muscle weakness 
 C. Pain relieved by sitting 
 D. Pain relieved by standing
A

Pain relieved by sitting

Causes of low back pain include vertebral disk herniation and spinal stenosis. Numbness and muscle weakness may be present in both. Pain in spinal stenosis is relieved by sitting and aggravated by standing. Pain from a herniated disk is aggravated by sitting and relieved by standing.

How well did you know this?
1
Not at all
2
3
4
5
Perfectly
51
Q

A 32-year-old white female marathon runner presents with persistent pain and tenderness over the pubic symphysis. With exercise, pain is noted in the lower abdomen and medial thighs. There is no fever. A radiograph shows widening of the pubic symphysis. What is the most likely diagnosis?

A

Osteitis pubis

Osteitis pubis is characterized by the chronic occurrences of pain in the lower abdomen and medial thighs with exercise. It produces tenderness over the pubic symphysis, and widening of the symphysis is often seen on plain radiographs. Pain from osteomyelitis is similar, but usually presents with fever and systemic symptoms, and does not cause widening of the symphysis. A stress fracture of the pubic ramus may occur in distance athletes, but the pain and tenderness are lateral, located over the posterior ramus. Inguinal hernia should not present with symphysis pain.

How well did you know this?
1
Not at all
2
3
4
5
Perfectly
52
Q

A 75-year-old female presents with a 1-month history of pain in her hips and shoulders bilaterally, accompanied by marked stiffness in the mornings. In addition, she reports a 4-lb weight loss and fatigue. She denies fever, chills, sweats, nausea, vomiting, swallowing disturbances, or changes in bowel habits. She specifically denies any visual symptoms or headache. Her physical examination is unremarkable, except for changes suggestive of osteoarthritis in the knees and hands. Laboratory testing reveals a hemoglobin level of 11.8 g/dL (N 13.0–16.0), a hematocrit of 36% (N 40–45), and an erythrocyte sedimentation rate of 84 mm/hr (N

A

Begin prednisone, 15 mg daily, with no plans for a temporal artery biopsy

It is generally not considered necessary to refer patients with classic polymyalgia rheumatica for a temporal artery biopsy in the absence of symptoms or signs of giant cell arteritis (e.g., headache, visual complaints, jaw claudication, fever, scalp tenderness, abnormal funduscopic exam). Corticosteroids should be started at relatively low doses (10–20 mg daily) and the patient followed for what should be a rapid clinical response.

How well did you know this?
1
Not at all
2
3
4
5
Perfectly
53
Q

What is recommended regarding the use of alendronate (Fosamax) for the treatment of osteoporosis in men?

A

Its effectiveness is similar to that seen in women

In men with hypogonadism, testosterone therapy has limited efficacy, and the efficacy of other therapies for osteoporosis in men has not been evaluated. Bisphosphonates, including alendronate, are indicated for treatment of Paget’s disease. They decrease bone pain, decrease bone resorption, and increase new bone formation. The proportion of men whose height decreased by at least 10 mm over a 2-year period was 13% in the placebo group and 3% in the alendronate group. The frequency of adverse gastrointestinal effects in the two groups was similar despite the fact that 36% of the men in the placebo group and 41% of those in the alendronate group reported taking NSAIDs during the study.

How well did you know this?
1
Not at all
2
3
4
5
Perfectly
54
Q
Which one of the following is associated with a reduced risk of postmenopausal osteoporosis? 
 A. Corticosteroid use 
 B. Cigarette smoking 
 C. Diuretic use 
 D. Low BMI 
 E. Asian ethnicity
A

Diuretic use

Diuretic use is associated with a lower risk of osteoporosis, perhaps due to decreased urinary calcium excretion. Cigarette smoking, low BMI, corticosteroid use, and Asian ethnicity are among the factors associated with increased risk.

How well did you know this?
1
Not at all
2
3
4
5
Perfectly
55
Q

The most common stress fracture in children involves which bone?

A

Tibia

Tibial fractures are the most common lower extremity stress fractures in both children and adults, accounting for about half of all stress fractures.

How well did you know this?
1
Not at all
2
3
4
5
Perfectly
56
Q

A 70-year-old white female with osteoporosis sees you for follow-up a few days after an emergency department visit for an acute T12 vertebral compression fracture. The fracture was suspected clinically and on plain films; the diagnosis was confirmed with a bone scan. The emergency department physician prescribed oxycodone (OxyContin) and NSAIDs, but the patient is still experiencing considerable discomfort. In addition to increasing the dosage of oxycodone, which one of the following interventions would you suggest now to reduce the patient’s pain?

A

Calcitonin (Miacalcin)

Calcitonin, either intranasal or subcutaneous, provides pain relief within a few days in many patients with osteoporotic vertebral compression fractures. The remainder of the choices do not provide acute relief. Vertebroplasty/kyphoplasty procedures are generally reserved for cases in which medical management has failed.

How well did you know this?
1
Not at all
2
3
4
5
Perfectly
57
Q

A 78-year-old white female presents with a 3-day history of lower thoracic back pain. She denies any antecedent fall or trauma, and first noted the pain upon arising. Her description of the pain indicates that it is severe, bilateral, and without radiation to the arms or legs. Her past medical history is positive for hypertension and controlled diabetes mellitus. Her medications include hydrochlorothiazide, enalapril (Vasotec), metformin (Glucophage), and a general multivitamin. She is a previous smoker but does not drink alcohol. She underwent menopause at age 50 and took estrogen for “a few months” for hot flashes. Physical examination reveals her to be in moderate pain with a somewhat stooped posture and mild tenderness over T12–L1. She has negative straight-leg raising and normal lower extremity sensation, strength, and reflexes. Which one of the following is true regarding this patient’s likely condition?

A

Subcutaneous or intranasal calcitonin (Calcimar, Miacalcin) may be very helpful for pain relief

The patient described has a classic presentation of an osteoporotic vertebral compression fracture. The diagnosis should be confirmed with a plain radiograph. Treatment is basically symptomatic, with a period of bed rest as short as possible (to avoid complications of immobility), pain medication, and bracing. Salmon calcitonin (injectable or intranasal) is often helpful in providing pain relief. Long-term management of underlying osteoporosis may help prevent future fractures.

How well did you know this?
1
Not at all
2
3
4
5
Perfectly
58
Q

A 59-year-old female with type 2 diabetes develops a 2x1-cm ulcer on the plantar aspect of her right foot. The ulcer is very deep and there is surrounding cellulitis. A plain film is normal. Which one of the following would be the imaging study of choice to rule out osteomyelitis in this patient?

A

An MRI scan

Although leukocyte scans are sensitive for the diagnosis of foot ulcers, MRI is now considered the imaging study of choice when osteomyelitis is suspected; the sensitivity and specificity of MRI in diabetic patients are 90% or greater.

How well did you know this?
1
Not at all
2
3
4
5
Perfectly
59
Q

A 38-year-old male is admitted to the hospital after being found lying on the floor unconscious from a drug overdose. The next morning, he is alert and complains of constant pain in the anterior aspect of his left leg. On examination, you note pain with passive stretching, pain with palpation anteriorly, and a slightly decreased dorsalis pedis pulse on the left. There is no edema or erythema. Radiographs of the lower extremity are normal. What should you do now?

A

Obtain immediate surgical consultation

The patient has a history and symptoms very suggestive of a compartment syndrome of the anterior compartment of the left leg. A common cause of this serious problem is limb compression during prolonged recumbency related to drug or alcohol overdose. Early diagnosis and treatment are essential in order to avoid permanent, severe disability. The five “P’s” of compartment syndrome are pain, pallor, paresthesias, pulselessness, and paralysis. If distal pulses are reduced, then muscle necrosis has occurred and immediate surgical consultation is necessary. Duplex doppler ultrasonography and impedance plethysmography are used to evaluate for deep venous thrombosis. Given the decreased pulses, however, this diagnosis is much less likely and such testing may needlessly delay urgent surgical treatment. Intravenous antibiotics would be appropriate for cellulitis. This diagnosis is unlikely in this scenario given the absence of fever, erythema, and edema, and the presence of a diminished pulse. Rest, ice, and elevation are inappropriate treatment modalities for compartment syndrome.

How well did you know this?
1
Not at all
2
3
4
5
Perfectly
60
Q

A football player sustains a finger injury in a preseason scrimmage. He reports getting his finger caught in another team member’s jersey. His right ring finger is tender on the volar surface, and is swollen around the distal interphalangeal joint. He is unable to flex at the distal interphalangeal joint, but can flex at the proximal interphalangeal joint and at the metacarpophalangeal joint. Radiographs do not show any bony abnormality. You would now:

A

Splint his finger in a slightly flexed position and urgently refer him to a hand surgeon

This injury is commonly called jersey finger. It is a tendinous rupture, sometimes with a bony avulsion fracture of the flexor digitorum profundus tendon. The tendon usually retracts, which makes nonsurgical treatment unlikely to result in complete healing. Splinting in hyperextension will distract the two ends of the tendon and it will not heal. A cast or flexion splint is unlikely to lend itself to complete healing because the tendon ends are unlikely to reattach to each other.

How well did you know this?
1
Not at all
2
3
4
5
Perfectly
61
Q

A 28-year-old white female complains of fatigue, stiffness, and a diffuse aching in her neck, shoulders, and back for several months. She is not sleeping well. A thorough physical examination is unremarkable except for tenderness over the upper trapezius, second costochondral junctions, lateral epicondyles, and medial knees. A CBC, erythrocyte sedimentation rate, and rheumatoid factor are unremarkable. What is the most appropriate management at this time?

A

Prescribe amitriptyline (Elavil), 10–25 mg every night at bedtime, along with exercise aimed at improving her overall level of fitness

This patient has a classic presentation of fibrositis-fibromyalgia syndrome. In addition to small bedtime doses of amitriptyline or cyclobenzaprine, symptoms are improved by an increase in physical fitness, stress reduction, regulation of sleep schedules, and reassurance. Opiate analgesics and sedative-hypnotics are not recommended for long-term treatment of this disorder.

How well did you know this?
1
Not at all
2
3
4
5
Perfectly
62
Q

A 2-year-old white male is seen for a well care visit. His mother is concerned because he is not yet able to walk. The routine physical examination, including an orthopedic evaluation, is unremarkable. Speech and other developmental landmarks seem normal for his age. Which tests would be most appropriate?

A

A serum creatine kinase level

The diagnosis of Duchenne’s muscular dystrophy, the most common neuromuscular disorder of childhood, is usually not made until the affected individual presents with an established gait abnormality at age 4–5. By then, parents unaware of the X-linked inheritance may have had additional children who would also be at risk. The disease can be diagnosed earlier by testing for elevated creatine kinase in boys who are slow to walk. The mean age for walking in affected boys is 17.2 months, whereas over 75% of normal children in the United States walk by 13.5 months. Massive elevation of creatine kinase (CK) from 20 to 100 times normal occurs in every young infant with the disease. Early detection allows appropriate genetic counseling regarding future pregnancies. Hypothyroidism and phenylketonuria could present as delayed walking. However, these diseases cause significant mental retardation and would be associated with global developmental delay. Furthermore, these disorders are now diagnosed in the neonatal period by routine screening. Disorders of amino acid metabolism present in the newborn period with failure to thrive, poor feeding, and lethargy. Gross chromosomal abnormalities would usually be incompatible with a normal physical examination at 18 months.

How well did you know this?
1
Not at all
2
3
4
5
Perfectly
63
Q

A 74-year-old African-American female has moderately severe pain due to osteoarthritis. However, she is also on medication for a seizure disorder. When choosing medications to manage her chronic pain, which one of the following should be used with caution because of her history of seizures?

A

Tramadol (Ultram)

According to the American Geriatrics Society 2002 clinical practice guidelines for management of persistent pain in older persons, tramadol has efficacy and safety similar to those of equianalgesic doses of codeine and hydrocodone. However, because of the threat of seizures (rare but potential), tramadol should be used with caution in patients with a history of seizure disorder or those taking other medications that lower seizure thresholds.

How well did you know this?
1
Not at all
2
3
4
5
Perfectly
64
Q

A 79-year-old white male with a previous history of prostate cancer has a lumbar spine film suggesting osteopenia. Subsequent bone density studies show a T score of –2.7. What would be appropriate therapy?

A

Alendronate (Fosamax)

The only approved treatments for male osteoporosis are alendronate and recombinant parathyroid hormone. Several drugs have been tested in clinical trials, and more pharmacologic treatments should become available in the future as male osteoporosis is increasingly recognized. Testosterone should not be used in this patient because of his history of prostate cancer.

How well did you know this?
1
Not at all
2
3
4
5
Perfectly
65
Q

A 28-year-old white male comes to your office complaining of pain in the right wrist since falling 2 weeks ago. On examination, he is tender in the anatomic snuffbox. A radiograph reveals a fracture of the proximal one-third of the carpal navicular bone (scaphoid).What is the most appropriate management at this time?

A

A thumb spica cast

Fracture of the scaphoid should be suspected in every “sprained wrist” where there is tenderness in the anatomic snuffbox. Radiographs may be negative initially. The scaphoid circulation enters the bone for the most part through the distal half. Fractures through the proximal third tend to cause loss of circulation and are slower to heal, and should be referred to an orthopedist because of the risk of nonunion and avascular necrosis. Fractures through the middle or distal one-third can be handled by the family physician in consultation with an orthopedist. The fracture is treated with a thumb spica cast for 10–12 weeks. A wrist splint does not provide adequate immobilization. A bone scan is unnecessary, and physical therapy is inappropriate. If there is still no evidence of union after 10 weeks of immobilization, the patient should be referred to an orthopedist for further care.

How well did you know this?
1
Not at all
2
3
4
5
Perfectly
66
Q

A 22-year-old male has acute low back pain without paresthesias or other neurologic signs. There is no lower extremity weakness. Which treatment has been shown to be of most benefit initially?

A

Resumption of physical activity as tolerated

Recent studies have shown superior benefits to allowing patients with acute low back pain, without sciatic involvement, to return to normal activities as tolerated. This was better than either bed rest or a back exercise program. Injections would be considered only if conservative therapy fails.

How well did you know this?
1
Not at all
2
3
4
5
Perfectly
67
Q

A 10-year-old male is brought to your office after sustaining a fall on an outstretched hand. Radiographs show a nondisplaced fracture of the middle third of the clavicle. Appropriate management would include:

A

A figure-of-8 splint or sling support

In treating the midshaft clavicular fracture, the goal is reduction of motion at the fracture site. This rarely requires operative intervention and can be managed by the family physician without orthopedic referral. The fracture site is best stabilized by restricting shoulder motion to less than 45 degree abduction. Either an arm sling or a figure-of-8 clavicular splint holding the shoulder back at the “position of attention” may be used. The figure-of-8 splint offers the advantage of leaving the elbow and hand free for activity. Ice and analgesics are used as needed during the acute stage of injury. Early use of heat may increase the inflammatory response. The patient may use the arm as pain permits, but should not abduct the arm more than 45 degrees for several weeks. The risk of adhesive capsulitis is negligible in children. Repeat radiographs at each follow-up office visit are not necessary, but a final radiograph should be ordered when clinical union has occurred to assess callus formation.

How well did you know this?
1
Not at all
2
3
4
5
Perfectly
68
Q

A 20-year-old white male presents to your office after a fall on an outstretched hand while skateboarding. He has pain at the anatomic snuffbox with no abrasion. Radiographs are negative. What would be the most appropriate management?

A

A thumb spica splint and follow-up radiographs in 2 weeks

This is a classic presentation of a possible scaphoid fracture. This fracture is important to diagnose and treat appropriately because of a high rate of non-union. If radiographs are negative, the patient should be placed in a thumb spica splint and have repeat radiographs in 2 weeks, because initial studies may be negative. An Ace bandage or a sugar tong splint would be inappropriate because they do not immobilize the thumb. A long arm cast for 8 weeks would immobilize the thumb, but could lead to loss of function, and may overtreat the injury if it is not truly a scaphoid fracture.

How well did you know this?
1
Not at all
2
3
4
5
Perfectly
69
Q

A 72-year-old white male with known coronary artery disease complains of pain in his back and legs which is increased by standing and walking and relieved by sitting. On examination, deep tendon reflexes in his legs are 0 to 1+ bilaterally. He has mild muscle weakness of his quadriceps and 1+ pedal pulses. He is taking ibuprofen, 800 mg three times a day, and using a back brace without much relief. What would be most likely to relieve his symptoms?

A

Posterior spinal decompression surgery

This patient’s symptoms are typical of spinal stenosis, as they are present when he is standing and relieved by sitting. He has already failed a trial of NSAIDs and bracing. Spinal decompression surgery is now indicated. Epidural corticosteroid injection might be helpful, but not trigger-point injections. Oral corticosteroids would be helpful if he had a herniated disc. Sympathectomy and bypass surgery are treatments for vascular occlusion.

How well did you know this?
1
Not at all
2
3
4
5
Perfectly
70
Q

A 62-year-old white male comes to your office with pain and swelling of the left great toe at the metatarsophalangeal joint. Examination shows it is erythematous, warm, swollen, and tender to touch. The patient has a history of diabetes mellitus controlled by diet, and hypertension. His medications include hydrochlorothiazide, 25 mg/day. A CBC and blood chemistry profile are normal, except for a uric acid level of 9.2 mg/dL (N 3.6–8.5). Which one of the following is true in this situation?

A

Stopping the hydrochlorothiazide may control the hyperuricemia

This is a typical presentation for gout. Elevated uric acid levels are not necessary for the diagnosis, as there are some patients with normal uric acid levels who still have gout. Conversely, hyperuricemia does not establish the diagnosis of gout. Risk factors for the development of gout include several enzyme deficiencies, renal insufficiency, hypertension, obesity, moonshine ingestion (causing lead exposure), and alcohol abuse. There are several medications that elevate uric acid, including diuretics, low-dose salicylates, niacin, cyclosporine, ethambutol, and pyrazinamide. A typical gout attack such as the one described will resolve spontaneously within 2 weeks without treatment. In patients who have an acute monoarticular arthritis in addition to gout, other diagnoses such as osteoarthritis, pseudogout, and infection must be considered. The diagnosis of gout is established by aspiration of synovial fluid or tophi, with characteristic uric acid crystals detected by polarized light microscopy. Treatment can consist of NSAIDs in healthy individuals. Indomethacin is considered the drug of choice. Corticosteroids can also be used, and are particularly helpful when the patient has renal insufficiency. Intra-articular injections of a corticosteroid such as triamcinolone are useful, and intramuscular corticosteroids may be especially useful in patients with polyarticular gout. Colchicine may be used, but may cause diarrhea. Life style changes such as weight loss, discontinuing alcohol use, and changing antihypertensive therapy is often all that is needed to control the hyperuricemia and thus prevent further attacks. Colchicine can be used for prophylaxis as well, although it does not alter hyperuricemia or prevent tophi from forming. If a patient has more than two attacks per year, urate lowering therapy is indicated. A 24-hour urine collection to identify whether the patient is an under-excretor or an overproducer of uric acid would indicate the correct medication. Overproducers are treated with allopurinol, while under-excretors benefit from probenecid if renal function is normal and there is no history of kidney stones.

How well did you know this?
1
Not at all
2
3
4
5
Perfectly
71
Q

A 21-year-old white female presents to the emergency department with a history consistent with a lateral ankle sprain that occurred 2 hours ago while she was playing softball. She complains of pain over the distal anterior talofibular ligament, but is able to bear weight. There is mild swelling, mild black and blue discoloration, and moderate tenderness to palpation over the insertion of the anterior talofibular ligament, but the malleoli are nontender to palpation. Which one of the following statements is true regarding the management of this case?

A

Early range-of-motion exercises should be initiated to maintain flexibility

This patient has an uncomplicated lateral ankle sprain and requires minimal intervention. The Ottawa ankle rules were developed to determine when radiographs are needed for ankle sprains. In summary, ankle radiographs should be done if the patient has pain at the medial or lateral malleolus and either bone tenderness at the back edge or tip of the lateral or medial malleolus, or an inability to bear weight immediately after the injury or in the emergency department, or both. If the patient complains of midfoot pain and/or bone tenderness at the base of the fifth metatarsal or navicular, or an inability to bear weight, radiographs should be ordered. Sprains can be differentiated from major partial or complete ligamentous tears by anteroposterior, lateral, and 30 degrees internal oblique (mortise view) radiographs. If the joint cleft between either malleolus and the talus is >4 mm, a major ligamentous tear is probable. Stress radiographs in forced inversion are sometimes helpful to demonstrate stability, but ankle instability can be present with a normal stress radiograph. Grade I and II ankle sprains are best treated with RICE (rest, ice, compression, elevation) and an air splint for ambulation. NSAIDs are used for control of pain and inflammation. Heat should not be applied. Early range-of-motion exercises should be initiated to maintain flexibility. Weight bearing is appropriate as tolerated and functional rehabilitation should be started when pain permits. Exercises on a balance board will help develop coordination.

How well did you know this?
1
Not at all
2
3
4
5
Perfectly
72
Q

The most serious complication of a slipped capital femoral epiphysis is:

A

Avascular necrosis

Avascular necrosis is the most serious complication of a slipped capital femoral epiphysis, and leads to more rapid arthritic deterioration. It may require hip fusion and total hip replacement early in adulthood.

How well did you know this?
1
Not at all
2
3
4
5
Perfectly
73
Q

A 39-year-old white male comes to your office with severe pain, fluctuance, erythema, and tenderness localized over the pad of the distal long finger. The most likely diagnosis is:

A

A felon (whitlow)

A felon, also called a whitlow, is an infection of the digital pulp of the terminal phalanx. The anatomic characteristics of this region compartmentalize infection, resulting in exquisite pressure and pain in the pulp spaces. As with all abscesses, incision and drainage is the single most important therapy. Paronychial and eponychial infections occur in the dorsal surface of the finger about the nail. Bacterial tenosynovitis is an extension of a deep space infection along the tendons. Digital cellulitis is a nonspecific term and does not adequately describe the infection depicted. The presence of fluctuance with this infection suggests abscess formation rather than a simple cellulitis.

74
Q

A 75-year-old Asian female comes to your office with a 6-week history of the gradual onset of localized low back pain. The pain is increased with walking or standing and relieved by sitting and lying. Pushing a shopping cart dramatically relieves the pain. Based on the history, which one of the following is the most likely diagnosis?

A

Spinal stenosis

Spinal stenosis pain is often referred to as pseudoclaudication because the pain is worsened by walking. Any movement that flexes the spine, such as pushing a shopping cart, relieves the pain. Pain from tumor or infection is persistent. Osteoporotic vertebral fractures have a sudden onset. Sciatica does not have the typical findings of spinal stenosis.

75
Q

A 3-year-old female is brought to your office for evaluation of mild intoeing. The child’s patellae face forward, and her feet point slightly inward. What would be most appropriate management?

A

Reassurance

Intoeing, as described, is usually caused by internal tibial torsion. This problem is believed to be caused by sleeping in the prone position, and sitting on the feet. In 90% of cases, internal tibial torsion gradually resolves without intervention by the age of 8. Avoiding prone sleeping enhances resolution of the problem. Night splints, orthotics, and shoe wedges are ineffective. Surgery (osteotomy) has been associated with a high complication rate, and is therefore not recommended in mild cases before the age of 8.

76
Q

A 9-month-old male is seen for a routine well-baby examination. There have been no health problems and developmental milestones are normal. Review of the growth chart shows that length, weight, and head circumference have continued to remain at the 75th percentile. The examination is normal with the exception of the anterior fontanelle being closed. Proper management at this time would include:

A

Serial measurement of head circumference

The anterior fontanelle in the newborn is normally 0.6–3.6 cm, with the mean size being 2.1 cm. It may actually enlarge the first few months, but the medial age of closure is 13.8 months. The anterior fontanelle closes at 3 months in 1% of cases, and by 1 year, 38% are closed. While early closure of the anterior fontanelle may be normal, the head circumference must be carefully monitored. The patient needs to be monitored for craniosynostosis (premature closure of one or more sutures) and for abnormal brain development. When craniosynostosis is suspected, a skull radiograph is useful for initial evaluation. If craniosynostosis is seen on the film, a CT scan should be obtained.

77
Q

A positive flexion abduction external rotation (FABER) test that elicits posterior pain indicates involvement of which joint?

A

Sacroiliac

When the flexion abduction external rotation (FABER) test elicits pain posteriorly, it indicates sacroiliac involvement. Anterior pain indicates hip involvement.

78
Q
Of the following, an 11-year-old who presents with knee pain is most likely to have:  
 A. Gout 
 B. Tibial apophysitis 
 C. A popliteal cyst 
 D. Inflammatory arthropathy 
 E. Pes anserine bursitis
A

Tibial apophysitis

The three most common knee conditions in children and adolescents are patellar subluxation, tibial apophysitis, and patellar tendinitis. Gout, osteoarthritis, and popliteal cysts present in older adults. Inflammatory arthritis is more common in adults than in children.

79
Q

What would be an indication for a second dose of pneumococcal polysaccharide vaccine (Pneumovax 23) in children?

A

Sickle cell disease

Patients with chronic illness, diabetes mellitus, cerebrospinal fluid leaks, chronic bronchopulmonary dysplasia, cyanotic congenital heart disease, or cochlear implants should receive one dose of pneumococcal polysaccharide vaccine after 2 years of age, and at least 2 months after the last dose of pneumococcal conjugate vaccine (Prevnar 13). Revaccination with polysaccharide vaccine is not recommended for these patients. Individuals with sickle cell disease, those with anatomic or functional asplenia, immunocompromised persons with renal failure or leukemia, and HIV-infected persons should receive polysaccharide vaccine on this schedule and should be revaccinated at least 5 years after the first dose.

80
Q

The intranasal live, attenuated influenza vaccine would be appropriate for which patient populations?

A

A 5-year-old female who is otherwise healthy

The live, attenuated influenza vaccine is an option for vaccinating healthy, nonpregnant individuals age 5-49 years. The vaccine is administered intranasally. It is not indicated in patients with underlying medical conditions, such as chronic pulmonary or cardiovascular disease, or in patients with a history of Guillain-Barré syndrome, pregnant patients, or children and adolescents who receive long-term aspirin or salicylate therapy. Patients with a history of hypersensitivity to eggs should not receive this vaccine.

81
Q

Patients with obstructive sleep apnea have an increased risk for

A

hypertension

Obstructive sleep apnea-hypopnea syndrome is defined as the presence of at least five obstructive events per hour with associated daytime sleepiness. It is present in 2%-4% of the population. The prevalence in men is almost three times that seen in premenopausal women and twice that of postmenopausal women. Other factors associated with an increased prevalence are obesity, older age, and systemic hypertension.

82
Q

A 32-year-old white male teacher is seen for a paroxysmal cough of 5 days duration. He tells you that a student in his class was diagnosed with pertussis 3 weeks ago. What would be the best treatment?

A

Azithromycin (Zithromax)

Macrolides are considered first-line therapy for Bordetella pertussis infection. Trimethoprim/sulfamethoxazole is considered second-line therapy.

83
Q

A 60-year-old female is admitted to the hospital with pneumonia 1 week after her discharge following elective colorectal surgery. Her initial stay was 5 days and she had no complications. She had no signs of infection until 2 days ago when she developed a temperature of 39.1°C (102.4°F), a cough with yellow sputum, and hypoxia. She has no abdominal pain or diarrhea. Her pulse rate is slightly elevated to 96 beats/min, and her blood pressure is unchanged from baseline. A chest radiograph confirms a left lingular infiltrate. Methicillin-resistant Staphylococcus pneumonia is rare in this institution. What would be the best initial treatment for this patient?

A

Ceftazidime sodium (Fortaz) and gentamicin

This patient has a significant pneumonia that requires the initiation of empiric antibiotics. It is important to remember that because this patient was recently in the hospital, the usual coverage for community-acquired pneumonia is not adequate. Health care-associated pneumonia is more likely to involve severe pathogens such as Pseudomonas aeruginosa, Klebsiella pneumoniae, and Acinetobacter species. Methicillin-resistant Staphylococcus aureus also is a consideration, depending on local prevalence. Of the antibiotic regimens listed, ceftazidime and gentamicin is the only choice that covers these organisms.

84
Q

A 22-year-old competitive cross-country skier presents with a complaint of not being able to perform as well as she expects. She has been training hard, but says she seems to get short of breath more quickly than she should. She also coughs frequently while exercising. A review of systems is otherwise negative. Her family history is negative for cardiac or pulmonary diseases. Her physical examination is completely normal, and pulmonary function tests obtained before and after bronchodilator use are normal. After you discuss your findings with the patient, she acknowledges that her expectations may be too high, but can think of no other cause for her problem. What would be the next reasonable step?

A

A trial of inhaled albuterol (Proventil) for exercise-induced bronchospasm

Exercise-induced bronchoconstriction (EIB) is a very common and underdiagnosed condition in athletes. It is defined as a 10% lowering of FEV1 when challenged with exercise. The exercise required to cause bronchoconstriction is 5-8 minutes at 80% of maximal oxygen consumption. EIB is much more common in high-ventilation sports, such as track and cross-country skiing. It is also more common in winter sports, because of the inspiration of cold, dry air. In some studies the incidence among cross-country skiers is as high as 50%, and 40% of those who have positive tests for bronchospasm are unaware of the problem. A physical examination, as well as pulmonary function tests at rest and before and after bronchodilators, will be normal unless there is underlying asthma. Among athletes with EIB, 10% will not have asthma. Bronchoprovocative testing can be ordered, but if it is not available a trial with an albuterol inhaler is reasonable.

85
Q

A 25-year-old Hispanic male comes to the emergency department with the sudden onset of moderate to severe right-sided chest pain and mild dyspnea. Vital signs are normal. A chest film shows a loss of markings along the right lung margins, involving about 10%-15% of the lung space. The mediastinum has not shifted. The best INITIAL treatment would be

A

oxygen supplementation and close observation

A small spontaneous pneumothorax involving less than 15%-20% of lung volume can be managed by administering oxygen and observing the patient. The pneumothorax will usually resorb in about 10 days if no ongoing air leak is present. Oxygen lowers the pressure gradient for nitrogen and favors transfer of gas from the pleural space to the capillaries. Decompression with anterior placement of an intravenous catheter is usually reserved for tension pneumothorax. Chest tube placement is used if observation is not successful or for larger pneumothoraces. Strict bed rest is not indicated.

86
Q

A 30-year-old white male complains of several weeks of nasal stuffiness, purulent nasal discharge, and facial pain. He does not respond to a 3-day course of trimethoprim/sulfamethoxazole (Bactrim, Septra). Follow-up treatment with 2 weeks of amoxicillin/clavulanate (Augmentin) is similarly ineffective. Of the following diagnostic options, which one is most appropriate at this time?

A

Coronal CT of the sinuses

This patient has a clinical presentation consistent with acute sinusitis. Failure to respond to adequate antibiotic therapy suggests either a complication, progression to chronic sinusitis, or a different, confounding diagnosis. The diagnostic procedure of choice in this situation is coronal CT of the sinuses, due to its increased sensitivity and competitive cost when compared with standard radiographs. Cultures of the nasal discharge give unreliable results because of bacterial contamination from the resident flora of the nose.

87
Q

A 60-year-old male with a right-sided pleural effusion undergoes thoracentesis. Analysis of the pleural fluid reveals a protein level of 2.0 g/dL and an LDH level of 70 U/L. His serum protein level is 7.0 g/dL (N 6.0-8.3) and his serum LDH level is 200 U/L (N 100-105). Based on these findings, which one of the following is the most likely diagnosis?

A

Heart failure

Pleural effusions may be exudates or transudates. The distinction is important for an accurate diagnosis and to help determine what further evaluations may be necessary. Lights criteria use ratios of fluid/serum values for protein and LDH. Pleural fluid/serum ratios greater than 0.6 for LDH and 0.5 for protein are indicative of exudates. In the scenario presented, both ratios are approximately 0.3; therefore, the fluid is a transudate. The list of causes for transudates is much shorter than for exudates. The vast majority of transudates are due to heart failure, with cirrhosis being the next most common cause. Once there is reasonable certainty that the fluid is a transudate, additional studies usually are not necessary. The other conditions listed result in exudative pleural effusions. Ref: Porcel JM, Light RT: Diagnostic approach to pleural effusion in adults

88
Q

A 6-year-old female presents with a 24-hour history of dry cough, malaise, and a temperature of 39.8°C (103.6°F). She received an influenza vaccination shot 7 days ago. A rapid influenza test is positive for influenza A. Which one of the following would be the best treatment option for this patient?

A

Oseltamivir (Tamiflu)

Amantadine and rimantadine are not recommended for the treatment of influenza A because of the development of resistance to these drugs. Resistance is not a problem with neuramidase inhibitors such as oseltamivir in immunocompetent patients. Zanamivir is not recommended for treatment of children under the age of 7. Although this child has recently received influenza vaccine, this is not a contraindication to drug therapy.

89
Q

A 12-year-old female has a cough and slight shortness of breath on a daily basis. She is awakened by the cough at least 3 nights per week. What would be the most appropriate treatment for this patient?

A

Inhaled corticosteroids daily

This patient has moderate persistent asthma. The preferred and most effective treatment is daily inhaled corticosteroids. A leukotriene inhibitor would be less effective. Oral prednisone daily is not recommended because of the risk of inducing adrenal insufficiency. Short- and long-acting β-agonists are not recommended as daily therapy because either can cause tachyphylaxis. They are considered rescue medications rather than preventive treatments.

90
Q

Good evidence supports which one of the following measures for reducing the risk of postoperative pulmonary complications after major abdominal surgery?

A

Use of an incentive spirometer

Use of an incentive spirometer and similar lung expansion techniques such as chest physiotherapy have been shown to have significant benefit for reducing postoperative complications. A course of preoperative corticosteroids has been shown to be beneficial for patients with COPD. Prolonged antibiotic prophylaxis and postoperative total parenteral nutrition have not been shown to have any benefit.

91
Q

What is the schedule for administering rotavirus vaccine?

A

The first dose should be given prior to 12 weeks of age

The rotavirus vaccine is an oral vaccine recommended for infants in a 3-dose schedule at ages 2, 4, and 6 months. The first dose should be given between 6 and 12 weeks of age, with additional doses given at 4 to 10-week intervals. The vaccine cannot be initiated after 12 weeks of age and should not be administered after 32 weeks of age. The first rotavirus vaccine was taken off the market in 1999 after an increased risk of intussusception was noted in infants when the first dose of the vaccine was given after 12 weeks of age. Because of this, the two vaccines licensed in 2006 carry the recommendation that the vaccine not be initiated in infants over 12 weeks of age. There is no data regarding safety and efficacy after this age.

92
Q

What is recommended for the treatment of cough and cold symptoms in children younger than 2 years of age?

A

Nasal saline with bulb suction

No medication available in the United States has been shown to effectively treat cough or cold symptoms in children younger than 2 years of age. However, many agents are commonly prescribed despite reports of numerous minor, and some serious, adverse effects. For this reason, the American Academy of Pediatrics and the American Academy of Family Physicians recommend using only nasal saline, bulb suction, humidified air, and good hydration in children younger than 2 years of age.

93
Q

A 66-year-old male smoker is being evaluated for a persistent cough and difficulty breathing. Spirometry confirms a fixed obstructive pathology with an FEV1 of about 50% of predicted for habitus and age. His oxygen saturation is 89%-90% on room air. Which one of the following would be most effective to prevent worsening of this patient’s condition?

 A. A combined inhaled corticosteroid and long-acting β-agonist 
 B. A long-acting anticholinergic agent 
 C. Long-term oral corticosteroids 
 D. Oxygen therapy 
 E. Smoking cessation
A

This patient has moderate to severe COPD. Smoking cessation is the single most important therapeutic intervention in patients with this condition and should be the priority of care. No existing medications have been shown to modify the long-term decline in lung function that is typical of COPD, but smoking cessation does prevent this decline. Long-term use of oxygen in COPD patients who also have chronic, severe hypoxia (

94
Q

A 67-year-old smoker with a history of pulmonary tuberculosis at 22 years of age presents with a 6-month history of increasing shortness of breath. On office spirometry his FVC is 60% of predicted, his FEV1 is 80% of predicted, and his FEV1/FVC ratio is 0.8. Which one of the following would be the most appropriate next step in his evaluation?

A

Refer to a pulmonary laboratory for static lung volume measurement and diffusion studies

Pulmonary function tests are usually classified as normal, compatible with a restrictive defect, or consistent with obstructive airway disease. In restrictive ventilatory processes, the FVC is decreased, the FEV1 is decreased or normal, and the absolute FEV1/FVC is >0.7. In obstructive airway problems, findings include a normal or decreased FVC, a decreased FEV1, and an absolute FEV1/FVC

95
Q

What is a significant side effect of varenicline (Chantix)?

A

Suicidal ideation

Education, support, and medications are all valuable tools in assisting patients with a smoking habit. Varenicline, a clinically effective smoking-cessation product, has been associated with patient mood changes following the initiation of therapy, including suicidal thoughts and aggressive and erratic behavior. The other problems listed have not been associated with varenicline use.

96
Q

A 40-year-old male respiratory therapist presents for a health examination prior to hospital employment. His history indicates that as a child he lived on a farm in Iowa, and his examination is unremarkable, but a chest radiograph shows that both lung fields have BB-sized calcifications in a miliary pattern. No other findings are noted. A PPD skin test is negative. The findings in this patient are most likely a result of

A

histoplasmosis

Asymptomatic patients in excellent health often present with this characteristic chest radiograph pattern, which is usually due to histoplasmosis infection, especially if the patient has been in the midwestern United States. Exposure to bird or bat excrement is a common cause, and treatment is usually not needed. This pattern is not characteristic of the other infections listed, although miliary tuberculosis is a remote possibility in spite of the negative PPD skin test.

97
Q

A 60-year-old male presents to the urgent care center with a fever and a productive cough. He has a 40 pack/year history of cigarette smoking. In addition to lobar pneumonia on a chest radiograph, there is an incidental finding of bilateral pleural plaques. Which one of the following is the most likely cause of this finding?

A

Asbestos

Development of pleural plaques is the most common pathologic pulmonary response to asbestos inhalation. Over time, collagen is deposited in the pleura and may calcify. Most plaques are asymptomatic, and there is no evidence that plaques transform into malignant lesions. Plaques occur in approximately 50% of persons with heavy and prolonged exposure to asbestos and, therefore, are a marker of asbestos exposure. This should alert the physician to follow the patient for development of more serious asbestos-related diseases (e.g., lung cancer and mesothelioma). Although the other substances listed are associated with pulmonary diseases (coal dust and silicon dust with pneumoconiosis, and vinyl chloride and radon with lung cancer), none is associated with pleural plaques as found in this patient.

98
Q

A 72-year-old male slipped on a rug in his kitchen and struck his right side against a counter. He presents several days after the fall with a complaint of ongoing pain in his flank. He has a history of chronic atrial fibrillation, which is treated with warfarin (Coumadin). His vital signs are normal. A physical examination reveals tenderness to palpation along the posterior-lateral chest wall and decreased breath sounds in the right base. Radiographs reveal two fractured ribs on the right side and a moderately large pleural effusion in the right hemithorax. Laboratory test results include a hemoglobin of 10.5 mg/dL (baseline 11.0-12.0 mg/dL) and a prothrombin time of 33.5 seconds with an INR of 3.5. Which one of the following would be the most appropriate management at this time?

A

Evacuation of the pleural space

This patient has been clinically stable despite losing what appears to be a fair amount of blood into his pleural space after fracturing two ribs, a condition referred to as hemothorax. The treatment of choice in this condition is to remove the bloody fluid and re-expand the associated lung. This therapy is felt to decrease any ongoing blood loss by having the lung pleura put a direct barrier over the site that is bleeding. It also prevents the development of empyema or fibrosis, which could occur if the blood were to remain.

99
Q

A 70-year-old male sees you because of slowly increasing problems with COPD. He has had frequent exacerbations requiring emergency department visits. He currently uses a tiotropium (Spiriva) inhaler once a day, as well as an albuterol (Proventil) inhaler, 2 puffs 4 times a day as needed. An examination shows decreased breath sounds throughout, and an oxygen saturation of 92%. Spirometry shows he has severe COPD (stage III); his FEV1/FVC ratio is 65% of predicted and his FEV1 is 45% of predicted. The most reasonable change in treatment would be to add

A

inhaled fluticasone (Flovent), 2 puffs twice daily

This patient is suffering from severe COPD (stage III) and has a history of frequent exacerbations. The addition of a corticosteroid inhaler for patients with severe disease has been found to significantly decrease the number of exacerbations, but has no effect on overall mortality. Side effects of oral candidiasis and easy bruising of the skin are increased. Continuous oxygen has been shown to improve overall mortality and endurance in patients with an oxygen saturation of 88% or less, but has not been shown to improve quality of life in those with mild hypoxemia or if used only at night. Oral prednisone has been shown to be effective when used to treat acute exacerbations, but when used on a chronic basis it is no more effective than corticosteroid inhalers. Chronic oral prednisone is also associated with significant side effects, and therefore is not generally recommended. Oral theophylline has not been shown to be of benefit in either preventing exacerbations or improving quality of life, and has significant side effects of gastrointestinal toxicity, seizures, and arrhythmias. It should be reserved for carefully selected patients only.

100
Q

A previously healthy 20-month-old female is brought to the urgent-care clinic during the evening with a barking cough. On examination her rectal temperature is 37.9°C (100.2°F), respiratory rate 18/min, heart rate 120 beats/min, and O2 saturation 94%. She has stridor, with mild substernal retractions only when her temperature was taken. Which one of the following would be most appropriate at this point?

A

Dexamethasone, 0.6 mg/kg orally or intramuscularly as a single dose

Croup is a syndrome most often caused by viruses, but can occasionally be of bacterial origin as in laryngotracheitis, laryngotracheobronchitis (LTB), laryngotracheobroncheopneumonia (LTBP), or laryngeal diphtheria. Mild croup is manifested by an occasional barking cough with no stridor at rest, and mild or absent intercostal retractions. Moderate croup presents with a more frequent barking cough, stridor with suprasternal and sternal retractions at rest, but no agitation. Severe croup includes more prominent inspiratory and expiratory stridor with agitation and distress. There is good evidence that corticosteroids produce significant improvement. The regimens studied most frequently have consisted of single-dose dexamethasone (0.6 mg/kg orally or intramuscularly), with some studies including up to four more doses over a 2-day period. Longer courses of corticosteroids have not proven to be more effective and may be harmful, leading to secondary infections. Racemic epinephrine by nebulization is indicated in severe croup. Antitussives and decongestants have not been studied and are not recommended. Antibiotics are indicated in LTB and LTBP, which can be diagnosed on the basis of crackles and wheezing on examination, or by an abnormal chest radiograph. Laryngotracheitis can sometimes be associated with a bacterial infection, but should be suspected only after a patient does not improve with corticosteroids and epinephrine.

101
Q

A 42-year-old female presents with a 2-day history of chest pain. She describes the pain as sharp, located in the right upper chest, and worsened by deep breathing or coughing. She also complains of shortness of breath. She was previously healthy and has no recent history of travel. Her vital signs are normal. A pleural friction rub is noted on auscultation of the lungs. The remainder of the examination is normal. An EKG, cardiac enzymes, oxygen saturation, and a D-dimer level are all normal. What would be most appropriate at this point?

A

A chest radiograph

This patient has pleurisy. Patients presenting with pleuritic chest pain may have life-threatening disorders, and pulmonary embolism, acute myocardial infarction, and pneumothorax should be excluded. While 5%-20% of patients with pulmonary embolism present with pleuritic chest pain, this patient has no risks for pulmonary embolism and the normal D-dimer level obviates the need for further evaluation. Moderate- to high-risk patients may need a helical CT scan or other diagnostic testing. An EKG and chest radiograph are recommended in the evaluation of acute/subacute pleuritic chest pain. The chest radiograph will exclude pneumothorax, pleural effusion, or pneumonia. An echocardiogram would not be indicated if the cardiac examination and EKG are normal. An antinuclear antibody level could be considered in recurrent pleurisy or if other symptoms or signs of lupus were present, but it would not be indicated in this patient. Most cases of acute pleurisy are viral and should be treated with NSAIDs unless the workup indicates another problem.

102
Q

A 24-year-old female with a past history of asthma presents to the emergency department with an asthma exacerbation. Treatment with an inhaled bronchodilator and ipratropium (Atrovent) does not lead to significant improvement, and she is admitted to the hospital for ongoing management. On examination she is afebrile, her respiratory rate is 24/min, her pulse rate is 92 beats/min, and oxygen saturation is 92% on room air. She has diffuse bilateral inspiratory and expiratory wheezes with mild intercostal retractions. Which one of the following should be considered in the acute management of this patient?

A. Chest physical therapy 
 B. Inhaled fluticasone/salmeterol (Advair) 
 C. Oral azithromycin (Zithromax) 
 D. Oral prednisone 
 E. Oral theophylline
A

Oral prednisone

Hospital management of acute exacerbations of asthma should include inhaled short-acting bronchodilators in all patients. Systemic corticosteroids are recommended for all patients admitted to the hospital. The efficacy of oral prednisone has been shown to be equivalent to that of intravenous methylprednisolone (SOR A). Oxygen should also be considered in most patients. Antibiotics are not recommended in the treatment of asthma exacerbations unless there is a comorbid infection. Inhaled ipratropium bromide is recommended for treatment in the emergency department, but not in the hospital (SOR A). Chest physical therapy and methylxanthines are not recommended in the treatment of acute asthma exacerbations.

103
Q

A male infant is delivered at 41 weeks gestation by spontaneous vaginal delivery. The amniotic fluid is meconium stained. Apgar scores are 7 at 1 minute and 7 at 5 minutes. The baby is noted to have respiratory distress from birth and is hypoxic by pulse oximetry. Respiration improves with supplemental oxygen, as does the hypoxia, but does not return to normal. What would most likely be seen on a chest radiograph?

A

Patchy atelectasis

The chest radiograph of a child with meconium aspiration syndrome will show patchy atelectasis or consolidation. If the child has a normal chest film and respiratory distress, a noncardiopulmonary source should be considered (i.e., a neurologic or metabolic etiology). The chest film of a child with transient tachypnea of the newborn will show a wet silhouette around the heart, diffuse parenchymal infiltrates, or intralobar fluid accumulation. Homogeneous opaque infiltrates with air bronchograms on a chest radiograph are seen with hyaline membrane disease.

104
Q

A male infant is delivered by cesarean section because of dystocia due to macrosomia. Apgar scores are 8 at 1 minute and 10 at 5 minutes. However, at about 1 hour of age he begins to have tachypnea without hypoxemia. A chest radiograph shows diffuse parenchymal infiltrates and fluid in the pulmonary fissures. The symptoms resolve without treatment within 24 hours. The most likely diagnosis is

A

transient tachypnea of the newborn

This child had transient tachypnea of the newborn, the most common cause of neonatal respiratory distress. It is a benign condition due to residual pulmonary fluid remaining in the lungs after delivery. Risk factors include cesarean delivery, macrosomia, male gender, and maternal asthma and/or diabetes mellitus. The other conditions listed cause neonatal respiratory distress, but do not resolve spontaneously. They also cause additional significant abnormal findings on physical examination and/or ancillary studies such as imaging and laboratory studies.

105
Q

You see a 9-month-old male with a 1-day history of cough and wheezing. He has previously been healthy and was born after an uncomplicated term pregnancy. He is up to date on his immunizations. On examination his temperature is 38.6°C (101.5°F) and his respiratory rate is 30/min. He has diffuse wheezing and his oxygen saturation on room air is 94%. Because it is midwinter, you obtain a swab for influenza, which is negative. A chest radiograph shows peribronchiolar edema. Appropriate management would include which one of the following?

A

Supportive care only

This child has a respiratory syncytial virus (RSV) infection. Supportive care is the mainstay of therapy. If the child can take in fluids by mouth and tolerate room air, outpatient management with close physician contact as needed is reasonable, especially in the absence of significant underlying risk factors. Routine use of corticosteroids is not recommended (SOR B). Although up to 60% of infants hospitalized for bronchiolitis receive corticosteroid therapy, studies have not provided sufficient evidence to support their use. Inhaled corticosteroids have not been shown to be beneficial, and the safety of high doses in infants is unclear. Supplemental oxygen should be administered if functional oxygen saturation (SpO2) persistently falls below 90% and can be discontinued when an adequate level returns (SOR C). Antiviral therapy for RSV bronchiolitis is controversial because of its marginal benefit, cumbersome delivery, potential risk to caregivers, and high cost (SOR B). Studies of ribavirin in patients with bronchiolitis have produced inconsistent findings. Palivizumab is a preventive measure, and is not used for treatment of the active disease. It may be considered in select infants and children with prematurity, chronic lung disease of prematurity, or congenital heart disease (SOR A). If used, it should be administered intramuscularly in five monthly doses of 15 mg/kg, usually beginning in November or December (SOR C).

106
Q

A 27-year-old male presents with what he thinks is a sinus infection. He has a 2-day history of right maxillary pain associated with nasal congestion and clear rhinorrhea. The only significant findings on examination are a low-grade fever and subjective tenderness with palpation over the right maxillary sinus. Which one of the following treatments is most supported by current evidence?

 A. Antihistamines 
 B. Oral decongestants 
 C. Topical vasoconstrictor sprays 
 D. Oral analgesics 
 E. Nasal lavage
A

Oral analgesics

Although oral antibiotics are overwhelmingly prescribed as initial treatment in acute sinusitis, it has been shown that the majority of acute illnesses are viral in origin and that 98% of cases will resolve spontaneously. Analgesics are considered the mainstay of therapy for acute sinusitis, according to evidence-based recommendations (SOR A). Other treatments should be considered if symptoms are prolonged (>7 days) or severe (two or more localizing symptoms or signs of serious bacterial complications). There is little evidence of effectiveness for antihistamines, oral decongestants, or vasoconstrictor sprays. There is also little evidence of effectiveness for nasal lavage in acute sinusitis, although it has an emerging role in chronic sinusitis.

107
Q

A 32-year-old African-American female presents with a 3-day history of fever, cough, and shortness of breath. She has been healthy otherwise, except for a sinus infection 2 months ago treated with amoxicillin. She does not appear toxic. A chest radiograph reveals an infiltrate in the right lower lobe, consistent with pneumonia. What would be the best choice for antibiotic treatment?

A

Levofloxacin (Levaquin)

For previously healthy patients with community-acquired pneumonia and no risk factors for drug resistance, a macrolide such as azithromycin is the preferred treatment (SOR A). Doxycycline is also acceptable (SOR C). Patients who have been treated with antibiotics within the previous 3 months should be treated with a respiratory fluoroquinolone (moxifloxacin, gemifloxacin, or levofloxacin) (SOR A). A β-lactam plus a macrolide is also an alternative (SOR A). The antibiotic chosen should be from a different class than the one used for the previous infection. These alternative treatments are also recommended for those with comorbidities such as chronic heart, lung, liver, or renal disease; diabetes mellitus; alcoholism; malignancies; asplenia; immunosuppressing conditions or use of immunosuppressing drugs; or other risk factors for drug-resistant Streptococcus pneumoniae infection (SOR A).

108
Q

A 5-month-old infant has had several episodes of wheezing, not clearly related to colds. The pregnancy and delivery were normal; the infant received phototherapy for 1 day for hyperbilirubinemia. He had an episode of otitis media 1 month ago. There is no chronic runny nose or strong family history of asthma. He spits up small amounts of formula several times a day, but otherwise appears well. His growth curve is normal. An examination is unremarkable except for mild wheezing. Which one of the following is the most likely diagnosis?

A

Gastroesophageal reflux

Gastroesophageal reflux is a common cause of wheezing in infants. At 5 months of age, most infants no longer spit up several times a day, and this is a major clue that the wheezing may be from the reflux. Also, there is no family history of asthma and the wheezing is not related to infections. Cystic fibrosis is more likely to present with recurrent infections and failure to thrive than with intermittent wheezing.

109
Q

A 40-year-old African-American female presents to your office complaining of a persistent dry cough for the last 3 months. This seemed to start with a “bad chest cold.” She went to an urgent-care facility 6 weeks ago and the physician prescribed albuterol (Proventil, Ventolin) by metered-dose inhaler empirically for presumed reactive airways disease. This did not help. Now she also reports dyspnea on exertion that is noticeable when walking uphill. She has been taking nitrofurantoin (Macrodantin) for chronic urinary tract infections, but has an otherwise negative history. She works as a legal secretary. On examination, she is tachypneic. There is no cyanosis or clubbing. Her lungs are clear. Her height is 160 cm (63 in) and her weight is 60 kg (132 lb). Office pulmonary function tests reveal a forced vital capacity (FVC) of 1.4 L (average 3.3) and a 1-second forced expiratory volume (FEV1) of 1.6 L (average 2.8). An inhaled bronchodilator produces no improvement in these numbers. Which one of the following is the most likely cause of her problem?

A

Interstitial lung disease

This patient has a markedly reduced FVC with an FEV1/FVC ratio of 1.14%. This is consistent with moderately severe pulmonary restriction. Most likely the patient has chronic interstitial restrictive lung disease. Nitrofurantoin can cause this picture, usually after continuous treatment for 6 or more months, and pulmonary function may be impaired permanently. A wide variety of additional causes have been described including noxious gases, pulmonary hypersensitivities, neoplasia, and systemic diseases (e.g., sarcoidosis). Management includes avoidance of the offending agent or treatment of the underlying condition.

110
Q

A 25-year-old white male who has a poorly controlled major seizure disorder and a 6-week history of recurrent fever, anorexia, and persistent, productive coughing visits your office. On physical examination he is noted to have a temperature of 38.3 degrees C (101.0 degrees F), a respiratory rate of 16/min, gingival hyperplasia, and a fetid odor to his breath. Auscultation of the lungs reveals rales in the mid-portion of the right lung posteriorly. What is most likely to be found on a chest radiograph?

A

A lung abscess

Anaerobic lung abscesses are most often found in a person predisposed to aspiration who complains of a productive cough associated with fever, anorexia, and weakness. Physical examination usually reveals poor dental hygiene, a fetid odor to the breath and sputum, rales, and pulmonary findings consistent with consolidation. Patients who have sarcoidosis usually do not have a productive cough and have bilateral physical findings. A persistent productive cough is not a striking finding in disseminated tuberculosis, which would be suggested by miliary calcifications on a chest film. The clinical presentation and physical findings are not consistent with a simple mass in the right hilum nor with a right pleural effusion.

111
Q

A healthy 24-year-old male presents with a sore throat of 2 days’ duration. He reports mild congestion and a dry cough. On examination, his temperature is 37.2 degrees C (99.0 degrees F). His pharynx is red without exudates, and there are no anterior cervical nodes. His tympanic membranes are normal, and his chest is clear. What’s the next appropriate step?

A

Treat with analgesics and supportive care

The Centers for Disease Control and Prevention (CDC) assembled a panel of national health experts to develop evidence-based guidelines for evaluating and treating adults with acute respiratory disease. In clinical screening, the most reliable predictors of streptococcal pharyngitis are the Centor criteria. These include tonsillar exudates, tender anterior cervical lymphadenopathy, absence of cough, and history of fever. The presence of three or four of these criteria has a positive predictive value of 40%–60%, and the absence of three or four of these criteria has a negative predictive value of 80%. Patients with four positive criteria should be treated with antibiotics, those with three positive criteria should be tested and treated if positive, and those with 0–1 positive criteria should be treated with analgesics and supportive care only. This patient has only one of the Centor criteria, and according to the panel should not be tested or treated with antibiotics.

112
Q

A 12-year-old white male asthmatic has an acute episode of wheezing. You diagnose an acute asthma attack and prescribe an inhaled beta2-adrenergic agonist, but despite 1–2 hours of treatment he continues to experience wheezing and shortness of breath. Which one of the following is the most appropriate addition to acute outpatient management?

A

Oral corticosteroids

The treatment of choice for occasional acute symptoms of asthma is an inhaled beta2-adrenergic agonist such as albuterol, terbutaline, or pirbuterol. However, acute symptoms that do not respond to beta-agonists should be treated with a short course of systemic corticosteroids. Theophylline has limited usefulness for treatment of acute symptoms in patients with intermittent asthma; it is a less potent bronchodilator than subcutaneous or inhaled adrenergic drugs, and therapeutic serum concentrations can cause transient adverse effects such as nausea and central nervous system stimulation in patients who have not been taking the drug continuously. Cromolyn can decrease airway hyperreactivity, but has no bronchodilating activity and is useful only for prophylaxis. Inhaled corticosteroids should be used for suppressing the symptoms of chronic persistent asthma. Oral beta2-selective agonists are less effective and have a slower onset of action than the same drugs given by inhalation.

113
Q

A 3-year-old child is brought to the emergency department with a barking cough, a hoarse voice, and mild stridor. You diagnose acute laryngotracheitis. Which treatment in the emergency department will reduce this child’s chance of subsequent hospitalization during this acute illness?

A

Dexamethasone (Decadron)

In laryngotracheitis (croup), intramuscular or oral dexamethasone (0.6 mg/kg) has been shown to reduce the rate of subsequent admission after acute treatment. Nebulized budesonide is also useful, but is not yet available in the United States. This disease is usually viral, and antibiotics are not helpful. Humidified air is of unproven benefit, and mist tents separate children from caregivers and hinder observation and evaluation.

114
Q

A healthy 68-year-old white male comes to your office for a health maintenance visit. He tells you he last saw a physician 6 years ago, and he recalls receiving a “flu shot” and a “pneumonia shot” at that time. According to current Centers for Disease Control (CDC) recommendations for giving pneumococcal vaccine (Pneumovax) in the elderly, this patient should be revaccinated:

A

Once

According to the most recent guidelines from the Centers for Disease Control, pneumococcal vaccine should be given to ALL persons 65 or over, including both previously unvaccinated persons and those who received the vaccine before age 65 but were vaccinated over 5 years ago. This patient was age 62 when he received the vaccine, indicating he should be revaccinated at the time of the health maintenance visit. Data from a single epidemiologic study suggest that the vaccine may provide protection for at least 9 years after receipt of the initial dose; however, data to support the need for subsequent doses of pneumococcal vaccine are not available, and because there is insufficient data concerning the safety of pneumococcal vaccine when administered three or more times, revaccination following a second dose is not routinely recommended.

115
Q

In a 1-year-old, pneumococcal 7-valent conjugate vaccine (Prevnar) is preferred rather than polyvalent pneumococcal vaccine (Pneumovax) because of which advantage?

A

It is more immunogenic

Pneumococcal 7-valent vaccine produces a satisfactory immune response in a 1-year-old, while polyvalent vaccine does not cause a good antibody response in children under the age of 2. Neither vaccine is available orally, and cost is not a factor. The 7-valent vaccine requires multiple doses. Neither vaccine can be combined with MMR.

116
Q

Pneumococcal 7-valent vaccine produces a satisfactory immune response in a 1-year-old, while polyvalent vaccine does not cause a good antibody response in children under the age of 2. Neither vaccine is available orally, and cost is not a factor. The 7-valent vaccine requires multiple doses. Neither vaccine can be combined with MMR.

A

A repeat PPD in 2 weeks

In 2000, the American Thoracic Society and the Centers for Disease Control and Prevention (CDC) advocated a shift in focus from screening the general population to testing only patients at increased risk for developing tuberculosis. In some persons PPD reactivity wanes with time but can be recalled by a second skin test administered 1 week or more after the first (i.e., two-step testing). For persons undergoing PPD skin testing, such as health-care workers, initial two-step testing may preclude misclassification of persons with boosted reactions as PPD converters. In those at low risk, such as this patient, a tuberculin skin test is now considered positive only if induration is at least 15 mm. Thus, this hospital volunteer would pose little risk to the hospital population since her 10-mm reaction falls within the guidelines of a negative test. She does not require diagnostic evaluation at this time, and isoniazid therapy is not indicated.

117
Q

What physical exam finding is an indicator of low risk for streptococcal infection in a patient with acute pharyngitis?

A

Cough

Indicators of a low risk of streptococcal infection include the absence of fever in patients not using antipyretic agents, the absence of pharyngeal erythema, and the presence of obvious manifestations of the common cold. Uncharacteristic symptoms include coryza, hoarseness, cough, and diarrhea.

118
Q

The most common illness related to occupational exposure is:

A

Asthma

While occupational exposure to large and small particles and gases is common and can lead to long-term pulmonary difficulties, asthma remains the most common illness related to on-the-job exposures. Bronchitis, obstructive pulmonary disease, and pulmonary fibrosis can all be associated with occupational exposure to various toxins, but the incidence is less than that of asthma. Ref: Beckett WS: Occupational respiratory diseases. N Engl J Med 2000

119
Q

A 10-year-old white male is brought to your office with a chief complaint of “head congestion” associated with moderate malaise and a low-grade fever for 7 days. He has had a thick, discolored nasal discharge for the last 2 days. What’s the appropriate next step?

A

No antibiotics should be used at this time

Clinical diagnosis of bacterial sinusitis requires the following: prolonged nonspecific upper respiratory signs and symptoms (i.e., rhinosinusitis and cough without improvement for >10–14 days), or more severe upper respiratory tract signs and symptoms (i.e., fever of 39 degrees C or higher, facial swelling, and facial pain). This individual does not meet these criteria, so antibiotics should not be used at this time. Although some believe that mucopurulent rhinitis (thick, opaque, or discolored nasal discharge) indicates the presence of bacterial sinusitis, this sign should be recognized as part of the natural course of a nonspecific, uncomplicated viral upper respiratory infection (URI). Sinus radiographs can demonstrate thickened mucosa, infundibular occlusion, and occasional air-fluid levels in uncomplicated viral URI.

120
Q

You are treating an 18-year-old white male college freshman for allergic rhinitis. It is September and he tells you that he has severe symptoms every autumn which impair his academic performance. He has a strongly positive family history of atopic dermatitis. Which medication is considered optimal treatment for this condition?

A

Intranasal glucocorticoids

Topical intranasal glucocorticoids are currently believed to be the most efficacious medications for the treatment of allergic rhinitis. They are far superior to oral preparations in terms of safety. Cromolyn sodium is also an effective topical agent for allergic rhinitis; however, it is more effective if started prior to the season of peak symptoms. Because of the high risk of rhinitis medicamentosa with chronic use of topical decongestants, these agents have limited usefulness in the treatment of allergic rhinitis. Some of the newer oral antihistamines have been found to be comparable in efficacy to intranasal steroids, but their use slightly increases the incidence of adverse effects and drug interactions. They are not as useful for congestion as for sneezing, pruritus, and rhinorrhea. Newer agents are relatively free of sedation. Overall, they are not as effective as topical glucocorticoids. Azelastine, an intranasal antihistamine, is effective in controlling symptoms but can cause somnolence and a bitter taste.

121
Q

You are caring for a 70-year-old male with widespread metastatic prostate cancer. Surgery, radiation, and hormonal therapy have failed to stop the cancer, and the goal of his care is now symptom relief. He is being cared for through a local hospice. Over the past few days he has been experiencing respiratory distress. His oxygen saturation is 94% on room air and his lungs are clear to auscultation. His respiratory rate is 16/min. Which medication would be best at this point?

A

Morphine

Dyspnea is a frequent and distressing symptom in terminally ill patients. In the absence of hypoxia, oxygen is not likely to be helpful. Opiates are the mainstay of symptomatic treatment.

122
Q

You diagnose sinusitis in a 7-year-old Hispanic male. He has been in good health otherwise, and has not been seen by a physician since his well care visit at age 6. What is recommended as initial antibiotic treatment?

A

Amoxicillin

In the absence of any risk factors, approximately 80% of children with acute bacterial sinusitis will respond to treatment with amoxicillin. Risk factors for the presence of bacterial species that are likely to be resistant to amoxicillin include attendance at day care, antimicrobial treatment within the past 90 days, and age

123
Q

At a routine annual visit, a healthy 65-year-old male is given pneumococcal vaccine. When should he receive a booster dose of this vaccine?

A

Never

If an otherwise healthy patient receives pneumococcal vaccine at age 65 or later, a second dose is not needed. If a healthy patient receives the vaccine before age 65, a second dose should be given after age 65 if at least 5 years have elapsed since the first dose.

124
Q

Residential and occupational exposure to radon is most closely linked with the development of:

A

Lung cancer

Radon is a naturally occurring, inert, radioactive gas which is a decay product of uranium. It can seep from soil beneath homes and reach concentrations in excess of the EPA standard of 4 picocuries. The EPA and the Surgeon General have recommended that all homes be tested for radon levels. Radon abatement measures can reduce radon concentration below EPA thresholds. Epidemiologic studies implicate radon as the second leading cause of lung cancer in the U.S., responsible for up to 30,000 of the 150,000 annual cases of bronchogenic cancer. Risk is heightened by concomitant exposure to tobacco smoke.

125
Q

What is the reason for the marked decline in the incidence of epiglottitis in the pediatric population?

A

Immunization with Haemophilus influenzae type b vaccine

The dramatic decrease in the incidence of epiglottitis is the result of routine vaccination with Haemophilus influenzae type b vaccine. This decreased incidence has also dramatically reduced morbidity and mortality.

126
Q

The mother of a 16-year-old male calls to report that her son has a severe sore throat and has been running a fever of 102 degrees F. What physical exam finding would be most specific for peritonsillar abscess?

A

Difficulty opening his mouth

Trismus is almost universally present with peritonsillar abscess. Voice change, otalgia, and odynophagia may or may not be present with peritonsillar abscess. Pharyngotonsillitis and peritonsillar cellulitis may also be associated with these complaints. Otalgia is common with peritonsillar abscess, otitis media, temporomandibular joint disorders, and a variety of other conditions. Peritonsillar abscess is rarely present without at least a 3-day history of progressive sore throat.

127
Q

A 7-year-old male presents with a 3-day history of sore throat, hoarseness, fever to 100 degrees (38 degrees C), and cough. Examination reveals injection of his tonsils, no exudates, and no abnormal breath sounds. What’s the appropriate next step?

A

Recommend symptomatic treatment (don’t have to do rapid strep test)

Pharyngitis is a common complaint, and usually has a viral cause. The key factors in diagnosing streptococcal pharyngitis are a fever over 100.4 degrees F, tonsillar exudates, anterior cervical lymphadenopathy, and absence of cough. Age plays a role also, with those

128
Q

A 7-year-old male with recurrent sinusitis has difficulty breathing through his nose. He has had chronic diarrhea and his weight is at the 5th percentile. Nasal polyps are noted on examination, in the form of grayish pale masses in both nares. No nasal purulence or odor is present. Which test should you order?

A

A sweat chloride test
This child has chronic diarrhea, recurrent sinusitis, and nasal polyps, and is underweight. Nasal polyps tend to occur more often in adult males, with the prevalence increasing in both sexes after age 50. Any child 12 years or younger who presents with nasal polyps should be suspected of having cystic fibrosis until proven otherwise. A sweat chloride test, along with a history and clinical examination, is necessary to evaluate this possibility. Nasal polyps are found in 1% of the normal population, but a full 18% of those with cystic fibrosis are afflicted. There is no association of polyps with Wilson’s disease, sarcoidosis, or emphysema, so serum ceruloplasmin, angiotensin-converting enzyme, and alpha1-antitrypsin levels would not be useful. An erythrocyte sedimentation rate likewise would yield limited information.

129
Q

An 84-year-old white female presents to your office with symptoms of an upper respiratory infection and a hacking cough. She admits to smoking one pack of cigarettes daily since she was 21 years of age. Which one of the following is true with regard to her tobacco use?

A

Sustained-release bupropion (Wellbutrin SR) has been shown to reduce the relapse rate for up to 12 months

Sustained-release bupropion has been shown to reduce the relapse rate for smoking cessation and blunt weight gain for 12 months. Beneficial effects of smoking cessation are seen even among older smokers. Evidence has now shown that smokers who switch to low-tar or low-nicotine cigarettes do not significantly decrease their health risks. The approved Food and Drug Administration medications for smoking cessation (sustained-release bupropion, nicotine patch, nicotine gum, nicotine inhaler, and nicotine nasal spray) have been shown to be safe and should be recommended for all patients without contraindications who are trying to quit smoking. The nicotine patch in particular is safe, and has been shown not to cause adverse cardiovascular effects. Ref: Fiore MC, Hatsukami DK, Baker TB: Effective tobacco dependence treatment. JAMA 2002

130
Q

Which one of the following is true concerning control of mild persistent asthma in the pediatric population?

A

Inhaled anti-inflammatory agents, such as glucocorticoids or cromolyn sodium, should be used initially to maintain chronic control of asthma

Initial medications for the control of moderately severe asthma in children should include an anti-inflammatory agent, such as glucocorticoids or cromolyn. Cromolyn may be used safely in any pediatric age group, including infants. Ipratropium bromide is useful in the treatment of COPD but has very limited use for asthma. Inhaled beta-adrenergic agents should be used every 4 hours if needed. Overuse of these agents has been associated with an increased mortality rate.

131
Q

The local school district has found that there is asbestos in many of the classrooms. The asbestos is contained in ceiling material used as acoustical insulation. Which one of the following statements is true regarding this situation?

A

Workers removing the ceiling material without respiratory protection would have an increased risk of mesothelioma

Asbestos removal from public buildings is not mandated by the Environmental Protection Agency unless there is severe and extensive damage and there are no other options such as special repairs or encapsulation that will control fiber release. Asbestos exposure has been associated with a variety of cancers, including lung cancer, mesothelioma, cancer of the oropharynx, cancer of the larynx, and gastrointestinal cancer. Cigarette smoking amplifies the effects of asbestos exposure. Smoking, drug and alcohol abuse, improper diet, and inadequate exercise are all greater public health hazards than asbestos. Asthmatics, especially when exercising, can be influenced by a variety of inhaled irritants, including sulfur dioxide and acidic aerosols in concentrations present in many cities, but not including asbestos.

132
Q

Sleep apnea is known to be associated with:

A

Hypertension

Sleep apnea is associated with hypertension, and may in fact be a cause of hypertension. Improvement of sleep apnea can be associated with improved blood pressure.

133
Q

A 35-year-old male consults you about a vague chest pain he developed while sitting at his desk earlier in the day. The pain is right-sided and was sharp for a brief time when it began, but it rapidly subsided. There was no hemoptysis and the pain does not seem pleuritic. His physical examination, EKG, and oxygen saturation are unremarkable. A chest film shows a 10% right pneumothorax. Which one of the following is true in this situation?

A

Outpatient observation with a repeat chest radiograph in 24 hours is indicated

The majority of patients presenting with spontaneous pneumothorax are tall, thin individuals under 40 years of age. Most do not have clinically apparent lung disease, and the chest pain is sometimes minimal at onset and may resolve within 24 hours even if untreated. Patients with small pneumothoraces involving less than 15% of the hemithorax may have a normal physical examination, although tachycardia is occasionally noted. The diagnosis is confirmed by chest radiographs. Studies of recurrence have found that an average of 30% of patients will have a recurrence within 6 months to 2 years. The treatment of an initial pneumothorax of less than 20% may be monitored if a patient has few symptoms. Follow-up should include a chest radiograph to assess stability at 24–48 hours. Indications for treatment include progression, delayed expansion, or the development of symptoms. The majority of patients with spontaneous pneumothoraces, and perhaps almost all of them, will have subcutaneous bullae on a CT scan.

134
Q

A health-care worker has a negative tuberculin skin test (Mantoux method). A second test 10 days later is positive. This result indicates:

A

Long-standing, latent infection

A positive result on the second, but the not the first, step of a two-step Mantoux tuberculin skin test indicates long-standing, latent infection.

135
Q

In adults, what is the most likely cause of chronic, unilateral nasal obstruction?

A

Nasal septal deviation

The most common cause of nasal obstruction in all age groups is the common cold, which is classified as mucosal disease. Anatomic abnormalities, however, are the most frequent cause of constant unilateral obstruction. Of these, septal deviation is the most common. Foreign body impaction is an important, but infrequent, cause of unilateral obstruction and purulent rhinorrhea. Mucosal disease is usually bilateral and intermittent. Adenoidal hypertrophy is the most common tumor or growth to cause nasal obstruction, followed by nasal polyps, but both are less frequent than true anatomic causes of constant obstruction.

136
Q

A 6-year-old male is brought in for evaluation by his mother, who is concerned that he may have asthma. She reports that he coughs about 3 days out of the week and has a nighttime cough approximately 1 night per week. There is a family history of eczema and allergic rhinitis. Which one of the following would be the preferred initial treatment for this patient?

A

A low-dose inhaled corticosteroid such as budesonide (Pulmicort Turbuhaler)

The National Asthma Education and Prevention Program (NAEPP) updated its recommendations for the treatment of asthma in 2002. Treatment is based on asthma classification. This child meets the criteria for mild persistent asthma: symptoms more than 2 times per week but less than once a day, symptoms less than 2 nights per month, peak expiratory flow (PEF) or FEV1 >80% of predicted, and a PEF variability of 20%–30%. Asthma controller medications are recommended for all patients with persistent asthma, and the preferred long-term controller treatment in mild persistent asthma is a low-dose inhaled corticosteroid. Cromolyn, leukotriene modifiers, nedocromil, and sustained-release theophylline are alternatives, but are not preferred initial agents. Quick-acting, quick-relief agents such as short-acting beta-agonists are appropriate for prompt reversal of acute airflow obstruction.

137
Q

The treatment of choice for a 4-month-old infant with suspected pertussis is:

A

Erythromycin

In spite of widespread vaccination of infants, pertussis occurs endemically in 3- to 5-year cycles in the U.S. It appears to be more common within populations not routinely immunized, such as Mennonite communities, but can occur widely. Infants younger than 6 months are affected most severely, although pertussis occurs in all age groups. The diagnosis is made by nasopharyngeal culture, but because the disease is uncommon and the organism is fastidious, laboratory personnel should be advised of the physician’s suspicion of pertussis. Treatment includes respiratory and nutritional supportive care, particularly for infants younger than 6 months. Antibiotic therapy is most effective in shortening the illness when given early, during the upper respiratory phase, but is indicated at any stage to reduce the spread of disease to others. The drug of choice is erythromycin, 40–50 mg/kg/day divided into four doses, for 14 days. Also effective are azithromycin and clarithromycin, which may be better tolerated with improved compliance. Resistance to these agents is rare. Penicillins and cephalosporins are ineffective. Gentamicin is potentially very toxic, and is not indicated.

138
Q

An 83-year-old male has a long history of COPD. His resting oxygen saturation is 86% on room air. Treatment includes oral bronchodilators, inhaled corticosteroids, inhaled beta-agonists, inhaled cholinergics, and home oxygen. Which one of his treatments has been shown to prolong survival in cases such as this?

A

Home oxygen

Treatment of hypoxemia is critical in the management of COPD and trials have shown a reduction in mortality with the use of oxygen for 15 or more hours daily. Inhaled beta-adrenergic agonists and cholinergic agents, either alone or in combination, provide symptomatic relief but do not prolong survival. Theophylline can be used for symptoms inadequately relieved by bronchodilators. Inhaled corticosteroids do not appear to alter the rate of decline in lung function in COPD. However, some evidence shows that these agents alleviate symptoms and reduce disease exacerbation. Pulmonary rehabilitation improves quality of life and reduces hospitalizations.

139
Q

What would be the most appropriate empiric therapy for nursing home–acquired pneumonia in a patient with no other underlying disease?

A

Levofloxacin (Levaquin)

The major concern with regard to pneumonia in the nursing-home setting is the increased frequency of oropharyngeal colonization by gram-negative organisms. In the absence of collectible or diagnostic sputum Gram’s stains or cultures, empiric therapy must cover Streptococcus pneumoniae, Staphylococcus aureus, Haemophilus influenzae, and gram-negative bacteria. Levofloxacin is the best single agent for providing against this spectrum of organisms.

140
Q

Which one of the following unimmunized patients should receive two doses of influenza vaccine?

A

A 5-year-old with asthma

Two doses of influenza vaccine are recommended for children under the age of 9 years unless they have been vaccinated previously. Children 3–8 years of age should receive one or two 0.5-mL doses of split-virus vaccine intramuscularly.

141
Q

A 50-year-old male who is a heavy smoker asks you about vitamin supplementation to prevent cancer and cardiovascular disease. The patient is unwilling to stop smoking. According to the 2003 recommendations of the U.S. Preventive Services Task Force, which one of the following is true regarding vitamin supplementation in adults who are middle-age or older?

A

Large supplemental doses of ~17beta-carotene may increase the risk of lung cancer in heavy smokers

The U.S. Preventive Services Task Force found that beta-carotene supplementation provides no benefit in the prevention of cancer in middle-aged and older adults. In two trials limited to heavy smokers, supplementation with beta-carotene was associated with a higher incidence of lung cancer and all-cause mortality. In general, little evidence was found to determine whether supplementation of any of the mentioned vitamins reduces the risk of cardiovascular disease or cancer.

142
Q

In the elderly, which measures of pulmonary function is characteristically increased?

A

Functional residual capacity

Although vital capacity declines with age, total lung capacity remains constant. The reduction in vital capacity results from an increase in residual volume. This increase in residual volume and functional residual capacity results from the collapse of small airways that occurs at higher lung volumes as age increases. Residual volume increases nearly 50% between early adulthood and age 70. In addition, standard spirometric measurements of lung function (i.e., forced expiratory volume in 1 second, peak expiratory flow rate, and maximal expiratory flow volume) have been shown to decline with age. Arterial oxygen tension also slowly declines with age.

143
Q

Which one of the following is true regarding PPD testing for tuberculosis?

A

Patients who have converted within the past year should be treated, regardless of age

Because the risk of developing active disease is highest in patients within 2 years after conversion, recent converters should generally be treated regardless of age. BCG vaccination has a limited effect on PPD reactivity; tests should not be interpreted any differently in patients who have previously received BCG. The use of a two-step approach (i.e., retesting 1–4 weeks later in patients who initially test negative) is designed to decrease the false-negative rate of PPD testing. The significance of a positive result on either phase of the test is the same. Patients who are HIV positive are at higher risk for false-negative PPDs and active disease, but PPD testing is not contraindicated.

144
Q

A 4-year-old child returns to your clinic after 3 days of treatment for acute otitis media. She had been taking amoxicillin, 40 mg/kg/day, and using acetaminophen and ibuprofen for pain and fever. On examination, the child still has a bulging red tympanic membrane on the left, with a flat tympanogram. She has continued to have a fever. Which antibiotic would be contraindicated?

A. Amoxicillin/clavulanate (Augmentin) orally
B. Ceftriaxone (Rocephin) intramuscularly
C. Cefuroxime (Ceftin) orally
D. Ciprofloxacin (Cipro) orally
E. Clindamycin (Cleocin) orally

A

Ciprofloxacin (Cipro) orally

All of these medications listed are indicated for second-line treatment of acute otitis media. Ciprofloxacin, however, is not approved for patients under age 18.

145
Q

You see a 90-year-old male with a 5-year history of progressive hearing loss. The most common type of hearing loss at this age affects:

A

Predominantly high frequencies

In the geriatric population, presbycusis is the most common cause of hearing loss. Patients typically have the most difficulty hearing higher frequencies such as consonants. Lower-frequency sounds such as vowels are preserved.

146
Q

A 4-year-old Hispanic female has been discovered to have a congenital hearing loss. Her mother is an 18-year-old migrant farm worker who is currently at 8 weeks’ gestation with her second pregnancy. The mother has been found to have cervical dysplasia on her current Papanicolaou (Pap) smear and has also tested positive for Chlamydia. The most likely cause of this child’s hearing loss is:

A

Cytomegalovirus

Cytomegalovirus (CMV) is the most common congenital infection and occurs in up to 2.2% of newborns. It is the leading cause of congenital hearing loss. The virus is transmitted by contact with infected blood, urine, or saliva, or by sexual contact. Risk factors for CMV include low socioeconomic status, birth outside North America, first pregnancy prior to age 15, a history of cervical dysplasia, and a history of sexually transmitted diseases. Infection can be primary or a reactivation of a previous infection. While the greatest risk of infection is during the third trimester, those occurring in the first trimester are the most dangerous to the fetus.

147
Q

Within a 2-week period you see an unusually large number of patients of all ages who complain of painless loss of central vision. You refer them appropriately to an ophthalmologist. The feedback you get is that most of these persons have bilateral central scotomas caused by an ophthalmoscopically visible macular defect. Which one of the following events would most likely explain this problem?

A

A solar eclipse

Direct observation of the sun without an adequate filter, which often occurs during a solar eclipse, results in a specific type of radiation injury termed solar (eclipse) retinopathy. The lens system of the eye focuses the sun’s light onto a small spot on the macula, usually in one eye only, producing a thermal burn. The resulting retinal edema may clear spontaneously with minor functional loss, or it may cause significant tissue atrophy, leaving a defect seen with an ophthalmoscope as a macular hole. This macular injury produces a permanent central scotoma. Visual blurring and difficulty with light perception are reversible manifestations of the tissue hypoxia associated with carbon monoxide poisoning. Dust and other particulate matter can produce injury to the cornea and conjunctiva. These lesions are painful, usually prompting medical attention and appropriate treatment. While severe contusions to the globe and periorbital structures can produce retinal detachment, the clinical vignette does not support a boxing-type injury. Although potatoes have eyes, there is no other significant relationship between potato salad and ophthalmologic disease.

148
Q

A 73-year-old white male who is otherwise in good health noted a sudden shower of flashing lights before his left eye a few hours ago. He has never had this symptom before. The most likely diagnosis is:

A

Retinal detachment

While all of the conditions listed are associated with flashing light before the eyes, a sudden shower of flashing lights is highly suggestive of retinal detachment. Migraine syndrome rarely begins in the eighth decade, vitreous floaters are more likely to be perceived as spots rather than lights, and central retinal vein thrombosis would likely cause more loss of vision. Emergency referral to an ophthalmologist is mandatory in cases of probable retinal detachment.

149
Q

Which one of the following is the most common risk factor for retinal detachment?

A

Posterior detachment of the vitreous

Vitreous detachment is very common after age 60 and occurs frequently in younger persons with myopia. The separation of the posterior aspect of the vitreous from the retina exerts traction on the retina, with the attendant risks of a retinal tear and detachment. Symptoms of retinal detachment may include light flashes (photopsia), a sudden appearance or increase in “floaters,” or peripheral visual field loss, any of which should prompt an ophthalmology referral. Cataract surgery can result in premature shrinkage of the vitreous and thereby poses an increased risk, but vitreous detachment resulting from other processes is more common. Hyphema, glaucoma, and diabetic retinopathy are not specific risk factors for retinal detachment.

150
Q

What is the most common cause of visual loss in children?

A

Amblyopia

Evaluation of visual symptoms in children can be challenging, but is important for identifying correctable conditions. Amblyopia, or “lazy eye,” is the most common cause of visual loss, with a prevalence of 2% in childhood. It is often related to strabismus, in which the image from one eye is suppressed in order to eliminate diplopia. Iritis is unusual and may have minimal symptoms; it is frequently associated with juvenile rheumatoid arthritis. Glaucoma does occur in children, often after cataract surgery. Eye trauma is relatively common, especially in boys. They may sustain abrasions, foreign bodies and penetrating injuries. Conjunctivitis will usually resolve without visual loss except when complicated by keratitis, such as in herpetic infections.

151
Q

A 16-year-old white female experiences the sudden onset of tender, bleeding gums, malodorous breath, and a bad taste in her mouth. On examination she is febrile and has cervical lymphadenopathy and tachycardia. Her mouth has marginal punched-out, crater-like depressions of the interdental gingival papillae and gingival margins that are covered with a gray pseudomembranous slough. The drug of choice for treating this condition is:

A

Penicillin

This patient has acute necrotizing ulcerative gingivitis. The onset is sudden and findings include tender, bleeding gums, malodorous breath, and a bad taste in the mouth. The disease is frequently associated with systemic findings such as fever, anorexia, malaise, tachycardia, cervical lymphadenopathy, and leukocytosis. Characteristic gingival lesions are seen, appearing as marginal punched-out, crater-like depressions of the interdental gingival papillae and gingival margin. They are covered with a gray pseudomembranous slough that is demarcated from the remainder of the gingiva and bleeds when removed. These lesions may be related to a single tooth, a group of teeth, or the gingiva throughout the mouth. Management includes removing predisposing factors such as stress, fatigue, heavy smoking, and poor nutrition that can cause tissue breakdown. Mouth rinses with warm half-strength hydrogen peroxide are useful. When fever and lymphadenopathy occur, antibiotic treatment is warranted and penicillin is the drug of choice. Tetracycline and erythromycin are good alternatives.

152
Q

A 25-year-old male has a dental infection associated with facial swelling and lymphadenopathy. Which one of the following is the most appropriate antibiotic?

A

Penicillin

Dental infections complicated by the development of cellulitis should be treated with oral antibiotic therapy. The antibiotic of choice is penicillin. Clindamycin should be used if a patient is allergic to penicillin.

153
Q

A 42-year-old African-American male recently traveled to the Caribbean for a scuba diving trip. Since his return he has noted brief intermittent episodes of vertigo not associated with nausea or vomiting. He is concerned, however, because these episodes occurred after sneezing or coughing and then a couple of times after straining while lifting something. He has had no hearing loss, and no vertigo with positional changes such as bending over or turning over in bed. The most likely cause of this patients vertigo is

A

a perilymphatic fistula

A perilymphatic fistula between the middle and inner ear may be caused by barotrauma from scuba diving, as well as by direct blows, heavy weight bearing, and excessive straining (e.g., with sneezing or bowel movements.) This patients recent trip involved two of these potential factors. Vestibular neuronitis is a more sudden, unremitting syndrome. Menieres disease is manifested by episodes of vertigo, associated with hearing loss and often with nausea and vomiting. Benign paroxysmal positional vertigo is more likely in older individuals, and is associated with postural change. Multiple sclerosis requires symptoms in multiple areas and is not thought to be provoked by climate change.

154
Q

What insulin regimens most closely mimics the normal pattern of pancreatic insulin release in a nondiabetic person?

A

Insulin detemir (Levamir) daily plus rapid-acting insulin with meals

Basal insulin provides a relatively constant level of insulin for 24 hours, with an onset of action in 1 hour and no peak. NPH gives approximately 12 hours of coverage with a peak around 6–8 hours. Regular insulin has an onset of action of about 30 minutes and lasts about 5–8 hours, with a peak at about 2–4 hours. New rapid-acting analogue insulins have an onset of action within 5–15 minutes, peak within 30–75 minutes, and last only about 2–3 hours after administration. Thus, a 70/30 insulin mix (typically 70% NPH and 30% regular) provides coverage for 12 hours, but the peaks of insulin release do not closely mimic natural patterns. NPH given twice daily along with an insulin sliding-scale protocol using regular insulin is only slightly closer than a 70/30 twice-daily regimen. Rapid insulin alone does not provide any basal insulin, and the patient would therefore not have insulin available during the night.

155
Q

In a patient with a severe anaphylactic reaction to peanuts, the most appropriate route for epinephrine is:

A

intramuscular

Intramuscular epinephrine is the recommended drug for anaphylactic reactions (SOR A). Epinephrine is absorbed more rapidly intramuscularly than subcutaneously.

156
Q

A 19-year-old college student comes to your office with significant pain in his right great toe that is making it difficult for him to walk. He has never had this problem before.
When you examine him you find increased swelling with marked erythema and seropurulent drainage and ulceration of the medial nail fold. The toe is very tender to touch, particularly when pressure is applied to the tip of the toe. The most appropriate initial management would be:

A

partial avulsion of the medial nail plate and phenolization of the matrix at this visit

This ingrown nail meets the criteria for moderate severity: increased swelling, seropurulent drainage, infection, and ulceration of the nail fold. In these cases, antibiotics before or after phenolization of the matrix do not decrease healing time, postoperative morbidity, or recurrence rates (SOR B). A conservative approach, elevating the nail edge with a wisp of cotton or a gutter splint, is reasonable in patients with a mild to moderate ingrown toenail who do not have significant pain, substantial erythema, or purulent drainage.

Either immediate partial nail avulsion followed by phenolization, or direct surgical excision of the nail matrix is effective for the treatment of ingrown nails (SOR B). Pretreatment with soaking and antibiotics has not been demonstrated to add therapeutic benefit or to speed resolution. Several studies demonstrate that once the ingrown portion of the nail is removed and matricectomy is performed, the localized infection will resolve without the need for antibiotic therapy. Bilateral partial matricectomy maintains the functional role of the nail plate (although it narrows the nail plate) and should be considered in patients with a severe ingrown toenail or to manage recurrences.

157
Q

A 59-year-old male who is morbidly obese suffers a cardiac arrest. Intravenous access cannot be obtained. Which one of the following is true regarding intraosseous drug administration in this patient?

A

There are no contraindications to intraosseous administration in this patient

The current American Heart Association ACLS guidelines state that intraosseous access can be obtained in almost all age groups rapidly, and is preferred over the endotracheal route. Any drug that can be administered intravenously can be administered intraosseously. Many drugs administered via an endotracheal tube are poorly absorbed, and drug levels vary widely.

158
Q

Ultrasonography shows a complete placenta previa in a 23-year-old primigravida at 20 weeks gestation. She has not experienced any vaginal bleeding. Which one of the following would be the most appropriate management for this patient?

A

Repeat the ultrasonography at 28 weeks gestation

Placenta previa is a relatively common incidental finding on second trimester ultrasonography. Approximately 4% of ultrasound studies at 20–24 weeks gestation show a placenta previa, but it occurs in only 0.4% of pregnancies at term, because of migration of the placenta away from the lower uterine segment. Therefore, in the absence of bleeding, the most appropriate management is to repeat the ultrasonography in the third trimester (SOR A).

Because many placenta previas resolve close to term, a decision regarding mode of delivery should not be made until after ultrasonography is performed at 36 weeks gestation. Digital cervical examinations should not be performed in patients with known placenta previa because of the risk of precipitating bleeding. Corticosteroids are indicated at 24–34 weeks gestation if the patient has bleeding, given the higher risk of premature birth. In patients with a history of previous cesarean delivery who have a placenta previa at the site of the previous incision, a color-flow Doppler study should be performed to evaluate for a potential placenta accreta. In such cases, MRI may be helpful to confirm the diagnosis.

159
Q

What class of diabetes medications acts primarily by stimulating pancreatic insulin secretion?

A

Sulfonylureas, such as glipizide (Glucotrol)

Biguanides and thiazolidinediones are insulin sensitizers that decrease hepatic glucose production and increase insulin sensitivity. Sulfonylureas and meglitinides stimulate pancreatic insulin secretion, while DPP-4 inhibitors prevent GLP-1 breakdown and slow the breakdown of some sugars. GLP-1 mimetics stimulate insulin secretion, suppress glucagon secretion, and promote β-cell production. Amylin analogs act with insulin to delay gastric emptying and they also inhibit glucagon release.

160
Q

A 30-year-old male presents with a 3-week history of severe, burning pain in his right shoulder. He recalls no mechanism of injury. An examination reveals weakness to resistance of the biceps and triceps, and with external rotation of the shoulder. Full range of motion of the neck and shoulder does not worsen the pain.
Which one of the following would be most likely to identify the cause of this patient’s problem?

A

Electromyography and nerve conduction studies

This patient has brachial neuritis, which can be difficult to differentiate from cervical radiculopathy, shoulder pathology, and cerebrovascular accident. The pain preceded the weakness, no trauma was involved, and the weakness is in a nondermatomal distribution, making brachial neuritis the most likely diagnosis. Electromyography is most likely to show this lesion, but only after 3 weeks of symptoms. MRI of the neck may show abnormalities, but not the cause of the current problem. Symptoms are not consistent with shoulder pathology, deep-vein thrombosis of the upper extremity, or cerebrovascular accident.

161
Q

A 30-year-old African-American female is being evaluated because of absent menses for the last 6 months. Menarche was at age 12. Her menstrual periods have frequently been irregular, and are accompanied only occasionally by dysmenorrhea. She had her first child 4 years ago, but has not been able to become pregnant since. A physical examination and pelvic examination are unremarkable. A serum pregnancy test is negative, prolactin levels are normal, and LH and FSH levels are both three times normal on two occasions.
These findings are consistent with:

A

ovarian failure

The history and physical findings in this patient are consistent with all of the conditions listed. However, the elevated FSH and LH indicate an ovarian problem, and this case is consistent with ovarian failure or premature menopause. Most pituitary tumors associated with amenorrhea produce hyperprolactinemia. Polycystic ovary syndrome usually results in normal to slightly elevated LH levels and tonically low FSH levels. Hypothalamic amenorrhea is a diagnosis of exclusion, and can be induced by weight loss, excessive physical exercise (running, ballet), or systemic illness. It is associated with tonically low levels of LH and FSH.

162
Q

A 30-year-old female presents to your office for an initial visit. She reports a long history of asthma that currently awakens her three times per month, necessitating the use of an albuterol inhaler (Proventil, Ventolin).
According to current guidelines, which one of the following would be optimal treatment?

A

Adding a low-dose inhaled corticosteroid

Inhaled corticosteroids improve asthma control more effectively in children and adults than any other single long-term controller medication (SOR A). This patient has mild persistent asthma and should be treated with a low-dose inhaled corticosteroid.

163
Q
A 26-month-old child presents with a 2-day history of 6–8 loose stools per day and a low-grade fever. When evaluating the child to determine whether he is dehydrated, which one of the following would NOT be useful? 
 A. Skin turgor 
 B. Capillary refill time 
 C. Respiratory rate and pattern 
 D. The BUN/creatinine ratio 
 E. The serum bicarbonate level
A

The BUN/creatinine ratio

The most useful findings for identifying dehydration are prolonged capillary refill time, abnormal skin turgor, and abnormal respiratory pattern (SOR C). Capillary refill time is not affected by fever and should be less than 2 seconds. Skin recoil is normally instantaneous, but recoil time increases linearly with the degree of dehydration. The respiratory pattern should be compared with age-specific normal values, but will be increased and sometimes labored, depending on the degree of dehydration.

Unlike in adults, calculation of the BUN/creatinine ratio is not useful in children. Although the normal BUN level is the same for children and adults, the normal serum creatinine level changes with age in children. In combination with other clinical indicators, a low serum bicarbonate level (

164
Q

A 70-year-old Asian male presents with hematochezia. He has stable vital signs. Lower endoscopy is performed, but is unsuccessful due to active bleeding.
What’s the appropriate next step at this point?

A

A technetium-99m blood pool scan

In most patients with heavy gastrointestinal bleeding, localizing the bleeding site, rather than diagnosing the cause of the bleeding, is the most important task. A lower GI series is usually nondiagnostic during heavy, active bleeding. A small-bowel radiograph may be helpful after the active bleeding has stopped, but not during the acute phase of the bleeding. A blood pool scan allows repeated scanning over a prolonged period of time, with the goal of permitting enough accumulation of the isotope to direct the arteriographer to the most likely source of the bleeding. If the scan is negative, arteriography would be unlikely to reveal the active source of bleeding, and is also a more invasive procedure. Exploratory laparotomy may be indicated if a blood pool scan or an arteriogram is nondiagnostic and the patient continues to bleed heavily.

165
Q

A patient presenting with severe carbon monoxide poisoning should be treated with:

A

oxygen

Patients with carbon monoxide poisoning should be treated immediately with normobaric oxygen, which speeds up the excretion of carbon monoxide.

166
Q

A patient who underwent coronary bypass grafting several months ago has been intolerant of all medications for cholesterol lowering. However, on the recommendation of a friend, he began taking red yeast rice that he purchased at a natural healing store. His cholesterol level has improved with this product and he has tolerated it so far.
You should consider monitoring which laboratory test in this patient, based on the active ingredient in red yeast rice?

A

Liver enzymes

Red yeast rice (Monascus purpureus) is a widely available dietary supplement that has been used as an herbal medication in China for centuries. In recent years it has been used for alternative management of hyperlipidemia in the U.S. Extracts of red yeast rice contain several active ingredients, including monacolin K and other monacolins, that have HMG-CoA reductase inhibitory activity and are considered to be naturally occurring forms of lovastatin. Red yeast rice extract lowers total cholesterol, LDL-cholesterol, and triglycerides. It may be useful for patients unable to tolerate statins due to myalgias, but requires periodic monitoring of liver enzymes because its metabolic effects and potential for consequences are similar to those of statins.

167
Q

A 40-year-old nurse presents with a 1-year history of rhinitis, and a more recent onset of episodic wheezing and dyspnea. Her symptoms seem to improve when she is on vacation. She does not smoke, although she says that her husband does. Her FEV1 improves 20% with inhaled β-agonists.
Which one of the following is the most likely diagnosis?

A

Occupational asthma

Occupational asthma merits special consideration in all cases of new adult asthma or recurrence of childhood asthma after a significant asymptomatic period (SOR C). Occupational asthma is often preceded by the development of rhinitis in the workplace and should be considered in patients whose symptoms improve away from work. Reversibility with β-agonist use makes COPD less likely, in addition to the fact that the patient is a nonsmoker. Cystic fibrosis is not a likely diagnosis in a patient this age with a long history of being asymptomatic. Sarcoidosis would be less likely to cause reversible airway obstruction and intermittent symptoms. Vocal cord dysfunction would not be expected to respond to bronchodilators.

168
Q

A previously healthy 60-year-old male is diagnosed with multiple myeloma after a workup for an incidental finding on routine laboratory work. He has no identified organ or tissue damage and is asymptomatic.
What would be appropriate treatment of this patient’s condition?

A

No treatment

This patient has smoldering (asymptomatic) multiple myeloma. He does not have any organ or tissue damage related to this disease and has no symptoms. Early treatment of these patients does not improve mortality (SOR A) and may increase the likelihood of developing acute leukemia. The standard treatment for symptomatic patients under age 65 is autologous stem cell transplantation. Patients over 65 who are healthy enough to undergo transplantation would also be appropriate candidates. Patients who are not candidates for autologous stem cell transplantation generally receive melphalan and prednisolone with or without thalidomide. Radiotherapy can be used to relieve metastatic bone pain or spinal cord compression.

169
Q

A 35-year-old male presents with a 4-month history of pain in the lower lumbar region without radiation. He works in retail sales, and the pain and stiffness prevent him from working. He estimates the pain to be 7 on a 10-point scale. He has been under the care of a chiropractor and has experienced some relief with spinal manipulation. His history is negative for red flags indicating a serious cause for his pain.
The only positive findings on a physical examination are diffuse mild tenderness over the lumbar region and mild limitation of lumbar mobility on forward and lateral flexion/extension maneuvers. Appropriate laboratory tests and imaging studies are all within normal limits.
In addition to appropriate analgesics, which one of the following modalities has the best evidence of long-term benefit in this situation?

A

Multidisciplinary rehabilitation

This patient has nonspecific chronic back pain, most likely a lumbar strain or sprain. In addition to analgesics (e.g., acetaminophen or NSAIDs) (SOR A) and spinal manipulation (SOR B), a multidisciplinary rehabilitation program is the best choice for management (SOR A). This program includes a physician and at least one additional intervention (psychological, social, or vocational). Such programs alleviate subjective disability, reduce pain, return the person to work earlier, and reduce the amount of sick time taken in the first year by 7 days. Benefits persist for up to 5 years. Back school, TENS, and SSRIs have been found to have negative or conflicting evidence of effectiveness (SOR C). There is no evidence to support the use of epidural corticosteroid injections in patients without radicular signs or symptoms (SOR C).

170
Q

A 55-year-old male presents to your office for evaluation of increasing dyspnea with exertion over the past 2 weeks. He has smoked 2 packs of cigarettes per day since the age of 20. He has had a chronic cough for years, along with daily sputum production. He was given an albuterol inhaler for wheezing in the past, which he uses intermittently. On examination he has a severe decrease in breath sounds, no evidence of jugular venous distention, no cardiac murmur, and no peripheral edema. A chest film shows hyperinflation, but no infiltrates or pleural effusion. Office spirometry shows that his FEV1 is only 55% of the predicted value.
You consider using inhaled corticosteroids as part of the treatment regimen for this patient. This has been shown to:

A

increase the risk of pneumonia

COPD has several symptoms, including poor exercise tolerance, chronic cough, sputum production, dyspnea, and signs of right-sided heart failure. The most common etiology is cigarette smoking. A patient with any combination of two of these findings, such as a 70-pack-year history of smoking, decreased breath sounds, or a history of COPD, likely has airflow obstruction, defined as an FEV1 ≤60% of the predicted value. In stable COPD, treatment is reserved for patients who have symptoms and airflow obstruction. Treatment options for monotherapy are all similar in effectiveness and include long-acting inhaled anticholinergics, long-acting β-agonists, and inhaled corticosteroids.

Inhaled corticosteroids will not reduce mortality or affect long-term progression of COPD. However, they do reduce the number of exacerbations and the rate of decline in the quality of life. There appears to be no increase in cataract formation or rate of fracture. These agents do have side effects, including candidal infection of the oropharynx, hoarseness, and an increased risk of developing pneumonia.

171
Q

Which one of the following patients is unlikely to benefit from vaccination against hepatitis A?
A. A missionary traveling to Mexico
B. A man who has sex with men
C. A methamphetamine addict
D. A patient with chronic hepatitis
E. A 40-year old recent immigrant from India

A

A 40-year old recent immigrant from India

Each of the individuals listed is at increased risk for hepatitis A infection or its complications, except for the Indian immigrant. Hepatitis A is so prevalent in developing countries such as India that virtually everyone is infected by the end of childhood, and therefore immune. Infection with hepatitis A confers lifelong immunity, so an adult from a highly endemic area such as India has little to gain from vaccination.

172
Q

For a healthy 1-month-old, daily vitamin D intake should be:

A

400 IU

It is now recommended that all infants and children, including adolescents, have a minimum daily intake of 400 IU of vitamin D, beginning soon after birth. The current recommendation replaces the previous recommendation of a minimum daily intake of 200 IU/day of vitamin D supplementation beginning in the first 2 months after birth and continuing through adolescence. These revised guidelines for vitamin D intake for healthy infants, children, and adolescents are based on evidence from new clinical trials and the historical precedent of safely giving 400 IU of vitamin D per day in the pediatric and adolescent population. New evidence supports a potential role for vitamin D in maintaining innate immunity and
preventing diseases such as diabetes mellitus and cancer.

173
Q

A 68-year-old male was seen in a local urgent-care clinic 6 days ago for upper respiratory symptoms and was started on cefuroxime (Ceftin). He presents to your office with a 2-day history of 4–5 watery stools per day with no blood or mucus. He is afebrile and has a normal abdominal and rectal examination. A stool guaiac test is negative, and a stool sample is sent for further testing.
What is the best initial management for this patient?

A

Stop the cefuroxime

This patient is at high risk for Clostridium difficile–associated diarrhea, based on his age and his recent broad-spectrum antibiotic use. The initial management is to stop the antibiotics. Treatment should not be initiated unless the stool is positive for toxins A and B. The recommended initial treatment for C. difficileenteritis is oral metronidazole. Probiotics may be useful for prevention, but their use is controversial. Loperamide should be avoided, as it can slow down transit times and worsen toxin-mediated diarrhea.

174
Q

A 74-year-old female presents with a several-month history of gradually increasing dyspnea on exertion, swelling in her feet and lower legs, and having to sleep sitting up due to increased shortness of breath while lying flat. She has been healthy otherwise, with no known heart disease or hypertension, and she has no significant family history of heart disease. An echocardiogram shows an ejection fraction of 20% and a thin-walled, diffusely enlarged left ventricle.
Which one of the following is the most likely diagnosis?

A

Dilated cardiomyopathy

This patient’s symptoms and echocardiographic findings indicate a dilated cardiomyopathy. In patients with hypertrophic cardiomyopathy the echocardiogram shows left ventricular hypertrophy and a reduction in chamber size. In restrictive cardiomyopathy, findings include reduced ventricular volume, normal left ventricular wall thickness, and normal systolic function with impaired ventricular filling. Arrhythmogenic right ventricular cardiomyopathy usually presents with syncope and without symptoms of heart failure, and segmental wall abnormalities would be seen on the echocardiogram. Highly trained athletes may develop echocardiographic evidence of eccentric cardiac hypertrophy, but no symptoms of heart failure would be present.

175
Q

Which one of the following is true regarding treatment of pressure ulcers?

A

Topical antibiotics should not be used for more than 2 weeks at a time

Trials have not definitively shown that nutritional supplements speed ulcer healing. The head of the bed should be elevated only as necessary, and should be kept to less than 30° to reduce shearing forces.Systemic antibiotics should only be used for cellulitis, osteomyelitis, and bacteremia. Topical antibiotics may be used for periods of up to 2 weeks (SOR C).

176
Q

The “Get Up and Go Test” evaluates for what?

A

Risk of falling

The “Get Up and Go Test” is the most frequently recommended screening test for mobility. It takes less than a minute to perform and involves asking the patient to rise from a chair, walk 10 feet, turn, return to the chair, and sit down. Any unsafe or ineffective movement with this test suggests balance or gait impairment and an increased risk of falling. If the test is abnormal, referral to physical therapy for complete evaluation and assessment should be considered. Other interventions should also be considered, such as a medication review for factors related to the risk of falling.

177
Q

A 20-year-old male presents with a complaint of pain in his right testis. The onset of pain has been gradual and has been associated with dysuria and urinary frequency. The patient has no medical problems and is sexually active. On examination he has some swelling and mild tenderness of the testis. The area posterior to the testis is swollen and very tender. He has a normal cremasteric reflex, and the pain improves with elevation of the testicle.
Which one of the following would be the most appropriate management of this patient?

A

Ceftriaxone (Rocephin) and doxycycline

This patient has epididymitis. In males 14–35 years of age, the most common causes are Neisseria gonorrhoeae and Chlamydia trachomatis. The recommended treatment in this age group is ceftriaxone, 250 mg intramuscularly, and doxycycline, 100 mg twice daily for 10 days (SOR C). A single 1-g dose of azithromycin may be substituted for doxycycline. In those under age 14 or over age 35, the infection is usually caused by one of the common urinary tract pathogens, and levofloxacin, 500 mg once daily for 10 days, would be the appropriate treatment (SOR C).
If there is concern about testicular torsion, urgent surgical evaluation and ultrasonography are appropriate. Testicular torsion is most common between 12 and 18 years of age but can occur at any age. It usually presents with an acute onset of severe pain and typically does not have associated urinary symptoms. On examination there may be a high-riding transversely oriented testis with an abnormal cremasteric reflex and pain with testicular evaluation. Color Doppler ultrasonography will show a normal-appearing testis with decreased blood flow.

178
Q

An elevation of serum alkaline phosphatase combined with an elevation of 5’-nucleotidase is most suggestive of conditions affecting which organ?

A

the liver

Alkaline phosphatase is elevated in conditions affecting the bones, liver, small intestine, and placenta. The addition of elevated 5’-nucleotidase suggests the liver as the focus of the problem. Measuring 5’-nucleotidase to determine whether the alkaline phosphatase elevation is due to a hepatic problem is well substantiated, practical, and cost effective (SOR C).

179
Q

A 60-year-old right-handed white male arrives in the emergency department with symptoms and signs consistent with a stroke. His past medical history is significant for tobacco abuse and chronic treated hypertension. He is alert and afebrile. His pulse rate is 100 beats/min, respirations 20/min, and blood pressure 190/95 mm Hg. He has a moderate right-sided hemiparesis and is aphasic. There are no other significant physical findings.

While appropriate tests are being ordered, immediate management in the emergency department should include which one of the following?

A

Monitoring oxygenation status with pulse oximetry

Maintaining adequate tissue oxygenation is an important component of the emergency management of stroke. Hypoxia leads to anaerobic metabolism and depletion of energy stores, increasing brain injury. While there is no reason to routinely administer supplemental oxygen, the potential need for oxygen should be assessed using pulse oximetry or blood gas measurement. Overzealous use of antihypertensive drugs is contraindicated, since this can further reduce cerebral perfusion. In general, these drugs should not be used unless mean blood pressure is >130 mm Hg or systolic blood pressure is >220 mm Hg. Antithrombotic drugs such as heparin must be used with caution, and only after intracerebral hemorrhage has been ruled out by baseline CT followed by repeat CT within 48–72 hours. Hypovolemia can exacerbate cerebral hypoperfusion, so there is no need to restrict fluid intake. Optimization of cardiac output is a high priority in the immediate hours after a stroke. Based on data from randomized clinical trials, corticosteroids are not recommended for the management of cerebral edema and increased intracranial pressure after a stroke

180
Q

Which one of the following is the most common secondary cause of nephrotic syndrome in adults?

A

Diabetes mellitus

Although most cases of nephrotic syndrome are caused by primary kidney disease, the most common secondary cause of nephrotic syndrome in adults is diabetes mellitus. Other secondary causes include systemic lupus erythematosus, hepatitis B, hepatitis C, NSAIDs, amyloidosis, multiple myeloma, HIV, and preeclampsia. Primary causes include membranous nephropathy and focal segmental glomerulosclerosis, each accounting for approximately one third of cases.

181
Q

A 19-year-old college student comes to your office with her mother. The mother reports that her daughter has frequently been observed engaging in binge eating followed by induced vomiting. She has also admitted to using laxatives to prevent weight gain.
What electrolyte abnormality is most likely to be found in this patient?

A

Hypokalemia

The patient described is likely suffering from bulimia. These patients use vomiting, laxatives, or diuretics to prevent weight gain after binge eating. This often causes a loss of potassium, leading to weakness, cardiac arrhythmias, and respiratory difficulty. The levels of other electrolytes are not as dramatically affected.